Study Guide
NEW YORK STATE POLICE
TROOPER EXAMINATION
Contents
Introduction .................................................................................................................................................. 1
How to Use this Study Guide .......................................................................................................................... 1
Important Note .......................................................................................................................................................1
Examination Overview ................................................................................................................................... 2
What Does the Examination Measure? ..................................................................................................................2
Taking the Examination ..........................................................................................................................................3
Cognitive Ability Measure .............................................................................................................................. 4
Reading ...................................................................................................................................................................4
Vocabulary ..............................................................................................................................................................7
Writing ................................................................................................................................................................. 10
Deductive Reasoning ........................................................................................................................................... 12
Ordering of Information ...................................................................................................................................... 15
Inductive Reasoning ............................................................................................................................................ 17
Pattern Recognition ............................................................................................................................................. 20
Grammar ............................................................................................................................................................. 22
Visualization ........................................................................................................................................................ 25
Spatial Orientation .............................................................................................................................................. 27
Flexibility of Closure ............................................................................................................................................ 31
Memorization ...................................................................................................................................................... 33
Additional Resources for Improving your Cognitive Abilities .............................................................................. 36
Personality Attributes Measure .................................................................................................................... 37
Sample Questions ................................................................................................................................................ 37
Personality Attributes Section Tips ..................................................................................................................... 37
Tips for the Day of the Examination .................................................................................................................... 39
General Test-Taking Tips .............................................................................................................................. 39
General Strategies for Taking the Examination ................................................................................................... 39
Responding to Questions .................................................................................................................................... 40
Strategies for Analyzing and Avoiding Errors ...................................................................................................... 41
Final Tips .............................................................................................................................................................. 42
After Taking the Practice Test ............................................................................................................................. 43
Answer Sheet ....................................................................................................................................................... 43
Examination Instructions ..................................................................................................................................... 44
Practice Test ................................................................................................................................................ 44
Using the Answer Sheet ...................................................................................................................................... 44
Memorization ...................................................................................................................................................... 44
Answer Key .......................................................................................................................................................... 81
Evaluating Your Performance .............................................................................................................................. 82
Introduction
NYSP Trooper Exam Study Guide 1
Copyright © 2020, IOS, Inc.
The purpose of this study guide is to enable you to efficiently and effectively prepare to participate in the
administration of the New York State Police Trooper Examination, a state-of-the-art test to identify those
individuals who will excel in careers with the New York State Police. The examination will be a critical hurdle in
the pathway toward achieving an employment opportunity as a trooper with the New York State Police. This guide
will provide you with critical information about the examination that will allow you to prepare for the test-taking
experience, hone your cognitive skills, minimize test-related anxiety and ultimately perform at your peak level.
Additionally, at the end of this study guide you will find a 65-question practice test. The practice test contains
questions that are similar to those you will find on the New York State Police Trooper Examination.
This guide was created specifically for the New York State Police Trooper Examination by the publisher of the
examination. No other source of information will be as informative or accurate concerning the content and
construction of the New York State Police Trooper Examination.
How to Use this Study Guide
This study guide will give you basic information about how to take the New York State Police Trooper Examination.
Read this information carefully so that you will know the instructions in advance. Thorough preparation for the
New York State Police Trooper Examination will increase your self-confidence; self-confidence, in turn, will
increase your chances of getting a high score.
This study guide will also discuss the different types of questions that will appear on the test. The skills and abilities
measured by the test will be defined. You will also find sample questions along with an answer key. Read the
definition of each skill or ability carefully and then try the sample questions. After you have answered the sample
questions, check your answers with the corresponding answer key.
You will also be provided with several important test-taking strategies. By following these tips, you will be able to
complete the New York State Police Trooper Examination more quickly and efficiently, and you will be able to
improve your overall performance on the exam.
Finally, at the end of this study guide there is a practice test. The practice test contains the same number of
cognitive ability items as the actual New York State Police Trooper Examination. For the best results and to get
the most out of the practice test, we recommend that you try to recreate the conditions you will be faced with on
test day as closely as possible. Various suggestions will be reviewed before you begin the practice test.
IMPORTANT NOTE
Do not memorize the sample questions and answers. Any question that has been released will not be used again.
You may run into questions that will be very similar, but you will not be tested with any of these exact questions.
These questions will give you good practice, but they will not give you the answers to any of the questions on your
exam.
Intro
ducti
on
Examination Overview
New York State Police Trooper Examination Study Guide 2
Copyright © 2022, IOS, Inc.
The New York State Police Trooper Examination will consist of 150 multiple-choice questions that will measure
your cognitive abilities as well as your behavioral characteristics and attitudes. The entire examination will last
two hours. The examination contains two sections: cognitive ability and personality attributes. Each of these
sections will be timed separately. You will be given one hour and forty minutes to complete the cognitive ability
section, and twenty minutes to complete the personality attributes section. Be sure to pace yourself so that you
can complete the entire exam. If you finish early, use the extra time to look over your work and check your
answers. There is no bonus for finishing early, so we recommend that you use all of your time to perfect your
exam.
WHAT DOES THE EXAMINATION MEASURE?
The New York State Police Trooper Examination was designed to measure numerous areas that are related to
successful performance as a New York State Police trooper. Specifically, the examination contains questions
covering two broad areas: cognitive ability and personality attributes. It is important to note that the examination
does not measure job knowledge or any specific skills that require specialized training. You will not need any
specific law enforcement knowledge or skills to succeed on the examination.
Cognitive Abilities
The New York State Police Trooper Examination assesses 12 narrow cognitive abilities, each classified under one
of five broad cognitive abilities:
Broad Cognitive Abilities
Narrow Cognitive Abilities
Reading/Writing
Reading
Writing
Fluid Intelligence/Reasoning
Deductive Reasoning
Ordering of Information
Inductive Reasoning
Pattern Recognition
Crystallized Intelligence
Grammar
Vocabulary
Visual Intelligence
Visualization
Spatial Orientation
Flexibility of Closure
Short-Term Memory
Memorization
These cognitive ability areas have been demonstrated to be essential for success in the New York State Police
trooper job based on job analytic research.
Personality Attributes
In addition to cognitive ability, the New York State Police Trooper Examination measures personality attributes
that are necessary for effective job performance as a trooper. For some items in this section, you will indicate your
level of agreement with presented statements. For other items in this section, you will choose which one of two
statements most closely reflects your beliefs or attitudes.
E
x
a
m
i
n
a
t
i
o
n
O
v
e
r
v
i
e
w
Examination Overview
New York State Police Trooper Examination Study Guide 3
Copyright © 2022, IOS, Inc.
TAKING THE EXAMINATION
The New York State Police has partnered with Pearson VUE Testing Centers to provide the exam at 250 locations
across the US, including 50 plus in New York State. The testing process is governed by many rules. Please
familiarize yourself with these rules and closely adhere to them so that you are not disqualified from the testing
process. To familiarize yourself with these rules, please review the New York State Police Candidate Handbook.
You may find the Candidate Handbook and other helpful resources on what to expect at Pearson VUE test centers
on New York State Polices Pearson VUE homepage: https://home.pearsonvue.com/nysp.
Code of Conduct / Test Security
The administration of the examination is a highly secure process. By participating in the testing process, you agree
that you will maintain the security and confidentiality of the test content with which you come in contact. This
means that you will not share information about the questions or answer options with anyone either during or
after the testing process.
All written test materials for this examination are the property of IOS. Candidates must not remove test material
from the test site or reproduce, reconstruct, or discuss the test content with others. Unauthorized possession or
disclosure of the test material is prohibited by law. Candidates found to have violated test security may be
disqualified from their examination(s) and may also be disqualified from taking any civil service examination for
five years. In some cases, violations may also result in disciplinary action, fine, or imprisonment.
Other actions may also lead to disqualification. Failure to follow written and oral instructions may also be grounds
for disqualification. Any use of a cell phone or communication device during the test may result in the candidate’s
disqualification.
Disqualification is based on state and local regulation and policy; specific circumstances are considered in
reviewing each incident when a disqualification is considered.
Security Reminders
Do not remove any test material from the test room and do not paraphrase, reconstruct, or reproduce the
test material in any way, either orally or in writing.
Do not discuss the test material with others. Not all candidates take the test on the same day, and you may
unknowingly pass along information to a candidate who has not yet taken the test.
Be careful to follow instructions and observe test security requirements before, during, and after the test.
Cheating
Test proctors will be present to instruct test takers and to monitor the security and fairness of the testing process.
If any candidate is determined to have in any way violated test rules or engaged in dishonest behavior, that
individual may be removed from the testing process by test proctors and may be disqualified. Any behavior that
demonstrates a willful violation of the test rules or dishonest behavior (e.g., viewing another test-taker’s answers,
talking with other test takers, sharing answers with other parties) may result in disqualification. You should seek
to avoid the appearance of any wrong-doing throughout the test-taking process. Keep your eyes on your testing
materials and do not engage in conversation with other test-takers during the test.
Test Etiquette
It is important that you be respectful to other test takers seated around you. Do not make loud noises or engage
in distracting behavior during the course of the test administration. If you use the washroom or exit the test
facility, do so quietly and with as little distraction as possible.
Cognitive Ability Measure
New York State Police Trooper Examination Study Guide 4
Copyright © 2022, IOS, Inc.
Cognitive abilities are capabilities of our brains to perceive, reason, understand, manipulate and interact with
stimuli in the world around us. Cognitive abilities are honed through education, training and practice. The New
York State Police Trooper Examination assesses twelve narrow cognitive abilities: reading, writing, deductive
reasoning, ordering of information, inductive reasoning, pattern recognition, grammar, vocabulary, visualization,
spatial orientation, flexibility of closure, and memorization. Understanding each cognitive ability, considering your
personal strengths and weaknesses, and allocating time to practice cognitive abilities that you tend to be weak in
is one of the best ways you can prepare for the examination.
In order to better familiarize you with the diversity of questions you will experience in the cognitive section of the
examination, this study guide will describe each cognitive ability area and the manner in which it might be
demonstrated on the job. These descriptions, along with sample questions, will help you prepare for the
examination. Your performance on the cognitive section of the examination can be dramatically improved by
reviewing these sample questions and the discussion section following each question regarding the proper
methods of solving each type of question.
READING
Reading is the ability to read written words and sentences and understand what is being communicated.
Troopers are required to read and understand a vast amount of training materials and are confronted with on-
going professional training throughout their careers that is presented in a written format. Troopers also have to
read and understand policies, directives and tactical plans.
The following question will allow you to assess your reading ability, identify any areas of weakness, and learn the
proper method for solving this type of question.
Sample question 1
Please use the following passage to answer Sample Question 1.
The State of New York voted to implement a new program designed to increase public preparedness for active
shooter events due to a rise in the number of these events per year. Due to a limited budget, the program was
not designed to reach all members of the public; instead, existing data on active shooter events were analyzed to
determine at-risk groups, and a target group was identified for training. A committee was formed to identify this
ideal target group, create a curriculum, and oversee the implementation of the training program.
The committee first considered available research, which led to the discovery that over 40% of active shooter
incidents throughout the last few years occurred in office buildings. The data also showed that 68.7% of active
shooter incidents ended in five minutes or less, while police tended to respond to these incidents within 10
minutes, on average. Based on this data, the committee theorized that training office workers on how to respond
to an active shooter event would have a large impact on public preparedness.
The committee surveyed members of the civilian workforce and state troopers in the Upstate New York region
regarding their preparedness for active shooter events. State troopers reported high levels of preparedness for
active shooter events, while the civilian workforce reported low levels of preparedness. Based on this survey and
their review of available research, the committee asserted that office workers would be an ideal target group for
the training program. Several large companies were then contacted to establish interest and seek participation
for a free on-site active shooter preparedness seminar.
Through a joint effort between the committee and the New York State Police, a curriculum was developed and
C
o
g
n
it
iv
e
A
bi
li
t
y
M
e
a
s
u
r
e
Cognitive Ability Measure
New York State Police Trooper Examination Study Guide 5
Copyright © 2022, IOS, Inc.
launched with the goal of informing office workers of current active shooter research and teaching them what to
do if an active shooter is present in the workplace. After its implementation, a follow-up survey showed a
significant increase in the levels of preparedness among office workers. The State of New York declared that the
program was a success and voted to expand the program to target other types of employees.
1.
The main idea of the passage is that _____.
a.
the number of active shooter victims may be reduced by educating the civilian population on what
to do if an active shooter is present in the workplace
b.
active shooter events impact office workers more than they impact workers in other industries
c.
a study revealed that state troopers were well trained on how to handle an active shooter scenario
d.
a program to increase preparedness for active shooter events among office workers was deemed
successful after its development and subsequent launch
Discussion
This question measures your reading ability, or your ability to read a lengthy passage and synthesize the theme of
that passage into a single summary statement. In order to do this, you have to identify what the author sought to
communicate and select the answer option that best summarizes that thought.
The best strategy to use in responding to reading questions, or any question that requires you to read a lot of
information, is to first review the question. Knowing what you are trying to answer will allow you to read the
passage with a keen eye. After you review the question, carefully read through the passage and keep in mind the
question that you are trying to answer. Take your time reading the passage. Rushing or skipping over sections will
only make it more difficult to arrive at the correct answer. Worse, you will probably end up having to read the
passage again. So, slow down and read for understanding the first time.
As you read each paragraph it is helpful to either underline key sentences that help summarize the paragraph or
make short notes in the margins to summarize the theme of each paragraph. Following are sample notes you
might keep to summarize each paragraph:
Paragraph 1: The State of New York formed a committee to identify a target group for a new
program to increase public preparedness for active shooter events.
Paragraph 2: The committee found existing research that showed a large percentage of active
shooter incidents occurred in office buildings, most incidents ended in five minutes or less, and
police respond to these incidents within 10 minutes.
Paragraph 3: The committee found that civilian workers reported low levels of preparedness for
active shooter events; combined with the existing research, they asserted that office workers
would be an ideal target group for the new program.
Paragraph 4: After its curriculum was developed, the program was launched and was declared a
success after levels of preparedness among office workers increased.
The question asks you about the main idea of the passage. By reviewing your summary notes, you can synthesize
the key information from the passage to arrive at a conclusion.
By combining the summary notes, we arrive at the following conclusion: a committee formed by the State of New
York (Paragraph 1) reviewed existing research and surveyed civilian workers before implementing an active
shooter preparedness program for office workers (Paragraphs 2 and 3); the program was launched and declared
Cognitive Ability Measure
New York State Police Trooper Examination Study Guide 6
Copyright © 2022, IOS, Inc.
a success (Paragraph 4). By knowing that these are the key points from the passage, you can systematically review
the answer options to see which one best fits.
Option “a” is incorrect because, while it is implied by the passage, it is never actually discussed that the number
of active shooter victims may be reduced by educating civilians. Additionally, this option is too specific and focuses
on a single fact that is implied by the passage; it is important to consider the passage as a whole when determining
the main idea.
Option “b” is incorrect because, like option “a,” it is never actually mentioned in the passage. While the passage
does state that “over 40% of active shooter incidents throughout the last few years occurred in office buildings,”
this does not mean that these events impact office workers more than they impact other workers. Also like option
“a,” this option is much too specific. The program’s focus on office workers is a key piece of information, but it
does not tell the entire story of the passage.
Option “c,” “a study revealed that state troopers were well trained on how to handle an active shooter scenario,”
is directly mentioned in the passage in Paragraph 3. However, like “a” and “b,” this option is too specific. It does
not take into account the information presented throughout the entire passage and instead focuses on a single
fact. Option “c” is therefore incorrect.
Option “d” fits the conclusion we produced from combining our summary notes about the passage. It mentions
the key points from the passage and summarizes them into a general conclusion about the passage: “a program
to increase preparedness for active shooter events among office workers was deemed successful after its
development and subsequent launch.”
Based on this analysis, you should select “d” as the correct answer.
Correct answer: D
Reading Tips
For all reading questions on the examination, you will be given a passage to read. You will then answer questions
that ask you to summarize the main idea of the passage and recall details from the passage. In order to answer
the questions correctly, you must understand the meaning of the preceding passage.
The most common error you are likely to make in responding to a question like this is to select an answer that
sounds correct, because the information comes from the passage, but is not relevant or accurate based on the
specific question that is asked. Your challenge is to carefully read and understand the passage and the question
that is asked. When you are asked about a passage’s main idea, be careful not to focus too much on specific facts
or pieces of information presented. Think about all of the important information presented and combine it to
produce a statement summarizing the entire passage.
All reading questions on the examination will contain a focal passage and questions that are highly similar to the
question you see here. To improve your reading and ultimately perform at your best on the NYSP Trooper
Examination, you should refer to Additional Resources for Improving Your Cognitive Abilities at the end of the
Cognitive Ability Measure section.
Cognitive Ability Measure
New York State Police Trooper Examination Study Guide 7
Copyright © 2022, IOS, Inc.
VOCABULARY
Vocabulary items measure the extent of words that you can understand in terms of their correct meanings.
When troopers read training materials, it is essential that they are able to understand the meanings of the words
presented to them in a written context. Additionally, because troopers must read and understand policies,
directives and tactical plans, they must be able to understand the definitions of the words they are reading in the
context in which they are presented.
Questions on the test will assess your vocabulary using two different formats. The first format will be associated
with a reading passage (see Sample Question 2). The second format will be presented with no context (see Sample
Question 3). The following questions will allow you to assess your knowledge of vocabulary, identify any areas of
weakness, and learn the proper method for solving these types of questions.
Sample question 2
Please use the following passage (this is the same passage that was presented in Sample Question 1) to answer
Sample Question 2.
The State of New York voted to implement a new program designed to increase public preparedness for active
shooter events due to a rise in the number of these events per year. Due to a limited budget, the program was
not designed to reach all members of the public; instead, existing data on active shooter events were analyzed to
determine at-risk groups, and a target group was identified for training. A committee was formed to identify this
ideal target group, create a curriculum, and oversee the implementation of the training program.
The committee first considered available research, which led to the discovery that over 40% of active shooter
incidents throughout the last few years occurred in office buildings. The data also showed that 68.7% of active
shooter incidents ended in five minutes or less, while police tended to respond to these incidents within 10
minutes, on average. Based on this data, the committee theorized that training office workers on how to respond
to an active shooter event would have a large impact on public preparedness.
The committee surveyed members of the civilian workforce and state troopers in the Upstate New York region
regarding their preparedness for active shooter events. State troopers reported high levels of preparedness for
active shooter events, while the civilian workforce reported low levels of preparedness. Based on this survey and
their review of available research, the committee asserted that office workers would be an ideal target group for
the training program. Several large companies were then contacted to establish interest and seek participation
for a free on-site active shooter preparedness seminar.
Through a joint effort between the committee and the New York State Police, a curriculum was developed and
launched with the goal of informing office workers of current active shooter research and teaching them what to
do if an active shooter is present in the workplace. After its implementation, a follow-up survey showed a
significant increase in the levels of preparedness among office workers. The State of New York declared that the
program was a success and voted to expand the program to target other types of employees.
2.
The underlined word “asserted” in the passage most closely means _____.
a.
implemented
b.
declared
c.
applied
d.
waived
Cognitive Ability Measure
New York State Police Trooper Examination Study Guide 8
Copyright © 2022, IOS, Inc.
Discussion
This question measures your ability to use context clues within a given passage to identify the meaning of a given
word. To the extent that you are familiar with a wide breadth of words and their meanings, you should be able to
recognize which answer option most closely defines the given word.
In order to answer this question, you should first find the given word within the passage. The underlined word
“asserted” can be found in the third sentence of the third paragraph. Read the sentence containing the underlined
word. You should attempt to determine the definition of the word based on the context in which it is presented.
Then read the answer options and see if the correct answer noticeably stands out to you.
If you cannot immediately identify the correct answer, replace the underlined word with each answer option and
read the sentence. Try to decide if the sentence makes sense with the answer option replacing the underlined
word. Eliminate answer options that you know are incorrect, and then make your best guess.
In this question, the definition of the underlined word “asserted is stated a fact or belief confidently and
forcefully.The definition of option “a,” implemented, is “put a decision or plan into effect.” The definition of
option “b,” declared, is “stated or made known formally or officially.” The definition of option c,” applied, is “put
into operation or practical use.” The definition of option “d,” waived, is “relinquished voluntarily.
In this question, the correct answer is b. Specifically, the word “declared” is the closest match to the word
“asserted.By utilizing the context of the sentence and your knowledge of vocabulary, you can conclude that the
correct answer is option “b.
Correct answer: B
Sample question 3
For Sample Question 3, choose the best answer.
3.
Identify the word that has the most similar meaning to “advocate.”
a.
Retract
b.
Authorize
c.
Reject
d.
Support
Discussion
This question measures your ability to identify a synonym, or a word similar in meaning, for a given word. To the
extent that you are familiar with a wide breadth of words and their meanings, you should be able to recognize
which answer option is most similar to the word provided.
In order to answer this question, you should first determine the definition of the given word. Then, you should
select the answer option that shares a similar definition. Read the answer options and see if the correct answer
noticeably stands out to you. If you cannot immediately identify the correct answer, carefully read through the
answer options and attempt to define each word. Eliminate answer options that you know are incorrect, and then
make your best guess.
In this question, the definition of the word “advocate” is “to support or argue for.The definition of option “a,”
retract, is “to take back or withdraw.” The definition of option b,” authorize, is “to endorse or permit.” The
Cognitive Ability Measure
New York State Police Trooper Examination Study Guide 9
Copyright © 2022, IOS, Inc.
definition of option “c,” reject, is “to refuse to accept.” Finally, the definition of option “d,” support, is “to assist
or help.”
In this question, the correct answer is d”. Specifically, the word supportis the closest match to the word
advocate.” By utilizing your knowledge of synonyms and vocabulary, you can conclude that the correct answer
is option “d.”
Correct answer: D
Vocabulary Tips
Some of the vocabulary items on the examination will ask you to identify the word with the most similar meaning
to a given word. Other vocabulary items will ask you to choose the correct definition of an underlined word within
a passage. This type of question will require you to be familiar with a variety of words and their meanings. The
more words you know, the better your chances of succeeding. Therefore, to increase your vocabulary knowledge,
you should consider referencing study aids or books developed for standardized test preparation. Additionally,
vocabulary building is easiest when you encounter words while reading. Whether it is books, magazines, or online
articles, written passages will often contain words that you are unfamiliar with. Whenever you come across a
word you are not familiar with, you should look up its definition. Doing this will help you expand your vocabulary
and become familiar with the definitions of a wider array of words.
To improve your vocabulary and ultimately perform at your best on the NYSP Trooper Examination, you should
also refer to Additional Resources for Improving Your Cognitive Abilities at the end of the Cognitive Ability Measure
section.
Cognitive Ability Measure
New York State Police Trooper Examination Study Guide 10
Copyright © 2022, IOS, Inc.
WRITING
Writing is the ability to communicate your thoughts using the English language in a way that other people will
understand.
Troopers are responsible for writing incident reports and communicating information through logs and records;
therefore, troopers must be able to communicate intelligently and professionally via the written word.
The following question will allow you to assess your writing ability, identify any areas of weakness, and learn the
proper method for solving this type of question.
Sample question 4
For the following item, please choose the summary that describes the most important details in the scenario most
clearly and accurately.
4.
Harrison Tucker recently bought a sailboat that he docks at the Brooklyn Marina. He is planning to
take his friends out sailing for his 30th birthday. Harrison stops by the marina the night before his
birthday to check on his boat and to drop off supplies for the next morning. When he arrives at the
marina, he hears shouting and loud music coming from one of the boats in the distance. As he walks
closer, Harrison is surprised to hear the noise is coming from his sailboat. He sees five strangers
dancing and drinking champagne on his boat. Harrison yells to the group that they must leave his
boat immediately. The group ignores Harrison and continues on with their party. Harrison calls the
police to ask for assistance in removing the partygoers from his sailboat, and the group is arrested
for trespassing.
a.
Harrison Tucker arrived at the Brooklyn Marina the night before his 30th birthday to find a group
of strangers dancing and drinking on his sailboat. Harrison told the strangers to leave before calling
the police; the police eventually arrested the group for trespassing.
b.
Harrison Tucker was dropping off supplies at the Brooklyn Marina the night before his 30th
birthday when he heard loud music and shouting coming from his sailboat in the distance. He
called the police and the group was arrested.
c.
Harrison Tucker saw five strangers dancing and drinking champagne on his sailboat. The group of
strangers ignored Harrison and continued partying when he told them to leave his boat
immediately.
d.
Harrison Tucker wanted to take his friends sailing to celebrate his 30th birthday. He arrived at the
Brooklyn Marina the night before to drop off supplies and saw a group of strangers dancing,
drinking, and playing music on his sailboat.
Discussion
Before you read the scenario, read the instructions carefully. This item requires you to select the choice that most
clearly and accurately presents the most important details described in the scenario. You can read the scenario
with this in mind, forming your own summary of the scenario as you read. It may be helpful to underline key points
within the scenario that you believe would be important to include in a summary, as in the following example:
Harrison Tucker recently bought a sailboat that he docks at the Brooklyn Marina. He is planning to take his
friends out sailing for his 30th birthday.Harrison stops by the marina the night before his birthday
1
to check on
his boat and to drop off supplies for the next morning. When he arrives at the marina, he hears shouting and
loud music coming from one of the boats in the distance. As he walks closer, Harrison is surprised to hear the
noise is coming from his sailboat. He sees five strangers dancing and drinking champagne on his boat. Harrison
Cognitive Ability Measure
New York State Police Trooper Examination Study Guide 11
Copyright © 2022, IOS, Inc.
yells to the group that they must leave his boat immediately.
2
The group ignores Harrison and continues on with
their party. Harrison calls the police
3
to ask for assistance in removing the partygoers from his sailboat, and the
group is arrested for trespassing.
4
After reading, you should have identified the following key pieces of information: 1) Harrison Tucker stops by
Brooklyn Marina the night before his birthday; 2) he sees strangers dancing and drinking champagne on his boat
and tells the group to leave; 3) the group ignores Harrison, so he calls the police; and 4) the group is arrested for
trespassing.
Options “b,” “c,” and “d” can be eliminated, as they are all missing several key pieces of information. Option “b”
fails to mention that Harrison told the group to leave prior to calling the police. Option “c” does not specify the
name of the marina, the date, or the fact that the group gets arrested for trespassing. Option “d” does not include
the fact that Harrison told the group to leave or that he called the police, who then arrested the group for
trespassing.
This leaves option aas the correct response. After reading option a, you will notice that it is the most accurate
and most clearly written statement. It mentions each key piece of information from the scenario: 1) Harrison
Tucker stops by Brooklyn Marina the night before his birthday; 2) he sees strangers dancing and drinking on his
boat and tells the group to leave; 3) the group ignores Harrison, so he calls the police; and 4) the group is arrested
for trespassing.
Correct answer: A
Writing Tips
For all writing items on the examination, you will be presented with a brief paragraph and asked to summarize it
in a single sentence. This type of question will assess your ability to use language economically and accurately.
You can practice this ability by reading a passage of text and attempting to produce a clear and concise summary
of the information presented. You should ensure that your summary does not skim over key details or include
irrelevant information. Ask a friend or family member to read your summary. If your summary clearly and concisely
describes the key information from the text, the reader should be able to determine what the original text was
about.
To improve your writing ability and ultimately perform at your best on the NYSP Trooper Examination, you should
also refer to Additional Resources for Improving Your Cognitive Abilities at the end of the Cognitive Ability Measure
section.
Cognitive Ability Measure
New York State Police Trooper Examination Study Guide 12
Copyright © 2022, IOS, Inc.
DEDUCTIVE REASONING
Deductive reasoning is the ability to apply rules and principles to make decisions about what to expect from a
specific situation. It involves applying general rules to specific problems to produce logical answers.
During training, troopers learn about criminal and traffic laws as well as the elements of crimes. They often have
to apply this information to specific situations that occur in the field. Applying state statutes, state policies,
procedures, and laws to specific situations in the field all involve deductive reasoning ability.
The following question will allow you to assess your deductive reasoning ability, identify any areas of weakness,
and learn the proper method for solving this type of question.
Sample question 5
Please use the information presented below to answer Sample Question 5.
Department Library Policies
Use of printed and audiovisual materials owned by the department:
The library location shall be housed in the administrative sector of the police department. All materials are
to be borrowed from and returned to this centralized area.
At least one set of study materials for all applicable promotional ranks shall always be kept on file. These
documents will be on “reserve” and shall not be removed or rented from the library. No exceptions.
From time to time, the department may issue printed items or textbooks to individuals for their use while
serving as members of the department.
Check out and return policy:
An individual may borrow books from the department library for a period of no more than 14 days. The
borrowed item must be returned to the library.
For promotional processes, all materials may only be borrowed for a maximum of 7 days.
An individual may check out a maximum of two books at one time.
To check out a book, the individual shall record the name of the item being borrowed on the Library Book
Checkout Log, along with the date and name of the borrower.
Books may NOT be loaned to another individual while checked out under any circumstances.
Audiovisual materials and equipment may also be checked out by a member of the department.
When an item is returned to the library, the borrower shall record the return date on the appropriate form.
Responsibilities:
The County Training Officer will be the custodian of the library. They will maintain the materials within the
library and regulate the use of the available resources.
Any individual who borrows or checks out materials from the department library shall be responsible for
that material entrusted to him/her.
Cognitive Ability Measure
New York State Police Trooper Examination Study Guide 13
Copyright © 2022, IOS, Inc.
5.
Tony borrowed a book on leadership from the department library. One day before the book is due,
he hands the book over to Alex who expressed interest in reading it next. According to the
information above, are Tony’s actions a violation of departmental policy?
a.
No, since Tony will be handing the book off to Alex a day before it’s due, he is not in violation of
keeping the book longer than outlined in the policy.
b.
No, since Alex is a member of the police department, he is eligible to directly take the book from
Tony once he has finished reading it.
c.
Yes, Tony must return the book to the library and record the return date on the appropriate form
to indicate the book has been returned. Alex can then rent the book under his own name.
d.
Yes, Tony must first check to ensure nobody else has placed a hold on the book before he can loan
it to Alex.
Discussion
Before you read the information in the passage, read the question carefully. This question is asking you to identify
whether Tony has violated departmental policy based on some specific facts described in the question. Knowing
that you are trying to identify whether a violation occurred, you can read the information contained in the passage
with this in mind. It may also helpful to highlight key words and descriptions within the passage that are in line
with the description presented in the question.
Based on the question, Tony borrowed a book from the library and then gave the book to Alex one day before it
was due. With this in mind, we can begin skimming the passage to find the answer to the question. The first
section, “Use of printed and audiovisual materials owned by the department,” does not contain any relevant
information.
The second section, “Check out and return policy,” seems to contain relevant information. In fact, the fifth point
states that “Books may NOT be loaned to another individual while checked out under any circumstances.” This
answers the question presented in the question stem, “are Tony’s actions a violation of departmental policy?”
The answer to this is “Yes.” Based on this, we can immediately eliminate answer options “a” and “b,” since they
both state that “No,” Tony’s actions were NOT a violation of departmental policy.
Having narrowed down our options to “c” and “d,” we must determine which of the two contains correct
information. Option “c” states that “Tony must return the book to the library and fill out the appropriate form to
indicate the book has been returned. Alex can then rent the book under his own name.Based on the last point
under “Check out and return policy,” it is true that when a book is returned to the library, the borrower must
record the return date on a form. It then follows that once the book is returned, Alex can rent it. Based on this
information, option “c” is correct. However, you should still review option “d” to ensure that it is not a better
answer.
Option “d” asserts that if no one has a hold on the book, Tony can loan it to Alex. This contradicts the fifth point
of “Check out and return policy,” which states that “Books may NOT be loaned to another individual while checked
out under any circumstances.” Option “d” is therefore incorrect. Option “c” must be the correct answer.
Correct answer: C
Cognitive Ability Measure
New York State Police Trooper Examination Study Guide 14
Copyright © 2022, IOS, Inc.
Deductive Reasoning Tips
For deductive reasoning questions, you will be asked to review a table or passage containing definitions, rules, or
policies. You will then be given a specific situation related to the information presented and have to determine
the best answer based on the question asked.
A common mistake made by candidates when responding to deductive reasoning questions is to attempt to
answer the question without carefully considering the information in the table or passage. Often candidates will
assume that they are familiar with the information in the table and will use their own knowledge to answer the
question. This might lead a candidate to answer the question without reading the full table or passage. Remember,
the information that is provided is all that is necessary to answer the question. Do not use prior knowledge of the
topic to determine the answer. If you do not use the information provided, there is a chance that you will answer
the question incorrectly.
The New York State Police Trooper Examination will contain questions like Sample Question 5 or questions that
reference information presented in tables. In each case, the information that is provided is essential for answering
the question. All such questions should be approached in the manner described here.
To improve your deductive reasoning and ultimately perform at your best on the NYSP Trooper Examination, you
should refer to Additional Resources for Improving Your Cognitive Abilities at the end of the Cognitive Ability
Measure section.
Cognitive Ability Measure
New York State Police Trooper Examination Study Guide 15
Copyright © 2022, IOS, Inc.
ORDERING OF INFORMATION
Ordering of information is the ability to identify the best or proper order of given actions or steps. It also involves
your ability to place events in a logical and meaningful order. This involves determining the order in which each
event in a scenario occurred to produce an accurate timeline. This ability is fundamental to understanding the
proper order of steps in performing a specific task safely.
Troopers often use ordering of information when they collect facts and accounts from witnesses and then attempt
to piece together the sequence of events based on disparate pieces of information. This ability is also used in
applying first aid, following a procedure in operating equipment or arranging sentences in a meaningful paragraph.
The following question will allow you to assess your ordering of information ability, identify any areas of weakness,
and learn the proper method for solving this type of question.
Sample question 6
Please use the passage below to answer Sample Question 6.
Walter Allen witnesses a traffic accident. His description of the accident includes the following six statements.
The statements are not in order.
1. The driver of the Suburban quickly approached the traffic jam, slammed on the brakes, and swerved
into the intersection to avoid hitting the vehicles.
2. The Suburban then continued through the intersection into oncoming traffic in the opposite lane.
3. I moved to the right lane so the Suburban could pass me, but then I noticed a large traffic jam ahead.
4. I called 911 when the vehicles collided. Both the sedan and Suburban were totaled.
5. When the Suburban entered the opposite lane, traffic on that side came to a halt, but only after the
Suburban was hit by a sedan.
6. I was driving on Route 59 when I noticed a black Suburban speed up very quickly behind me.
6.
Which of the following is the correct order of the above statements?
a.
6, 3, 1, 2, 5, 4
b.
6, 3, 2, 5, 1, 4
c.
6, 5, 3, 2, 1, 4
d.
6, 3, 2, 1, 5, 4
Discussion
In order to answer this question, you must first read through all six statements. When reading through them, you
may wish to make notes by each step indicating that it comes earlier or later in the process. If you identify the two
or three steps that come earlier, the two or three steps in the middle, and the two or three steps that come later,
you can then go back and rank-order these statements in smaller groups. This takes a complex task and breaks it
into more manageable pieces.
These statements provide a sequence of events that took place during a traffic accident. After reading through all
the statements, you will notice that the statements in total can describe the scene prior to the accident, during
the accident and just after the accident. Based on this analysis, you can make some quick notes to attempt to
place the statements into meaningful groups. Statements 1, 2, and 3 must have occurred before the accident,
Cognitive Ability Measure
New York State Police Trooper Examination Study Guide 16
Copyright © 2022, IOS, Inc.
because we know from skimming the subsequent statements that the Suburban was involved in the accident. You
should write “before” next to these first three statements. Statement 4 occurred after the accident, because the
witness indicates that both the sedan and Suburban were totaled. You should write “after” next to statement 4.
Statement 5 occurred during the accident, because it details exactly how the collision happened. Finally,
statement 6 occurred before the accident, as it informs us that the Suburban was speeding.
You will note that you have four statements that come before the accident, one during the accident and one after
the accident. Since your goal is to place these events in chronological order, you can immediately designate
statement 4 as the last event because it is the only one to have occurred after the accident. You can also
immediately designate statement 5 as the second to last event, as it is the only one that occurred during the
accident. This quick assessment allows you to immediately dismiss answer options band “cbecause they do
not list statement 5 as the second to last event.
Looking at the statements that come before the accident, statements 1, 2, 3, and 6, you should attempt to order
these chronologically. You will notice that statement 6 is likely the first event in the series because this statement
has the witness first noticing the speeding Suburban behind him. Because the Suburban is speeding up behind the
witness, it would make sense that the next event would have the witness switching lanes so the Suburban could
pass. Statement 3 fits as the second event. Statement 1 would be the third event because it has the Suburban
approach the traffic jam mentioned in statement 3 and swerve into the intersection. This statement also points
to the fourth statement as the suburban continues through the intersection into oncoming traffic. This leaves
statement 2 to be the fourth in the series. So, of these first four statements, we can organize them as follows: 6,
3, 1, and 2. The entire sequence is 6, 3, 1, 2, 5, 4. Option ais therefore the correct response based on your
organization of the six statements.
Once you have determined the proper sequence, read the statements through in order and make sure that the
sequence makes logical sense. If so, you have found the appropriate solution.
Correct answer: A
Ordering of Information Tips
The most common mistake that is made in ordering of information questions is failing to read all the steps before
beginning the organization process. You cannot determine the order of statements, or even categorize them
effectively, until you have considered all the choices. Another common mistake is failing to review the steps in
order after you have drawn a conclusion regarding their order. This last check ensures that the sequence is logical.
Often you will find that it makes sense to switch one or two of the steps around to allow the sequence to flow
better.
The ordering of information items on the New York State Police Trooper Examination will all follow the same
format as Sample Question 6. All such questions should be approached in the manner described here.
To improve your ordering of information ability and ultimately perform at your best on the NYSP Trooper
Examination, you should refer to Additional Resources for Improving Your Cognitive Abilities at the end of the
Cognitive Ability Measure section.
Cognitive Ability Measure
New York State Police Trooper Examination Study Guide 17
Copyright © 2022, IOS, Inc.
INDUCTIVE REASONING
Inductive reasoning is the ability to combine separate pieces of information, or specific answers to problems, to
form general rules or conclusions.
Troopers frequently exercise this ability when they take in multiple pieces of information about a crime scene or
incident and then make decisions about how to react to a scenario based on that information. This ability is used
when performing any type of investigation to conclude that rules or laws have been violated.
The following question will allow you to assess your inductive reasoning ability, identify any areas of weakness,
and learn the proper method for solving this type of question.
Sample question 7
Please use the information presented to answer Sample Question 7.
Trooper Web has been called to the scene of a road-rage incident. Four individuals provide information to Trooper
Web. Their accounts are below:
Marvis Smith: “I was sitting in my car on the side of the road when I heard a loud crunching sound. I looked up to
see that a red truck had rear-ended a white sedan. I saw both drivers exit their vehicles and proceed to have a
shouting match. Then, the driver of the truck got back into his vehicle and began to drive right toward me. He
nearly struck a pedestrian and another car with his truck before completely losing control and colliding into a
different car across the street. I thought about offering to help, but it turned out that the pedestrian who was
nearly hit was an off-duty state trooper.
Hailey House: “I was studying for my final exams at a coffee shop on Plymouth Avenue and Harvard Drive when I
saw an ambulance pass by with lights and sirens. I’m a volunteer EMT, so I exited the shop to investigate and saw
what appeared to be a multi-car accident about two blocks down. Upon my arrival the scene, a paramedic and an
off-duty state trooper were treating a man near a red truck. He didn’t appear to be very cooperative; I would
guess that he was heavily intoxicated.”
Travis Quin: I was walking down Warrenville Road when the accident occurred. I don’t really remember who was
at fault, but I think the sedan hit the truck. Then both drivers exited their vehicles and had a shouting match. The
truck driver got back into his vehicle and began to drive off. The truck came flying onto Plymouth Avenue, almost
hit a man, and then hit another car before stopping. I heard a volunteer EMT say that the truck driver was
intoxicated; I wouldn’t be surprised at all if it were true.”
Andrew Mavros: “I was stopped at the intersection of Washington Avenue and Grand Street when I heard a loud
crack. I turned left onto Washington Avenue toward the sound, which I believed to be from a car accident. As I
neared the Plymouth Avenue intersection, I saw that a red truck had rear-ended a white sedan. I figured I would
find somewhere to park and offer my assistance. As I exited my vehicle I was nearly run over by the truck as the
driver attempted to flee the scene. I watched him hit another car and then stop, so I went over to help him.
7.
Based on the witness statements, which of the following conclusions is TRUE?
a.
The truck driver was intoxicated.
b.
The initial accident was caused by the white sedan.
c.
Andrew Mavros is the off-duty state trooper.
d.
The truck driver attempted to hit a pedestrian.
Cognitive Ability Measure
New York State Police Trooper Examination Study Guide 18
Copyright © 2022, IOS, Inc.
Discussion
This question measures inductive reasoning, or your ability to combine information to arrive at a conclusion about
what happened. In this case, you need to consider the unique perspectives of four different individuals and use
them to weave together one cohesive picture.
The question asks you to determine which of the four conclusions presented is true. In order to answer this
question, you have to read the entire passage and use the details provided by each witness to determine what
happened. You should take notes as you read each individual’s account to make it easier to combine the
information they provide. Below are some example notes you may come up with as you read each individual’s
account:
Marvis Smith:
saw red truck rear-end white sedan
drivers had a shouting match
truck driver returned to vehicle and almost
hit an off-duty trooper
truck driver collided into another car across
street
Hailey House:
saw an ambulance from the coffee shop at
Plymouth Avenue and Harvard Drive
is a volunteer EMT
saw a paramedic and an off-duty trooper
treating a man near a red truck at the scene
guesses that the man was intoxicated
Travis Quin:
walking on Warrenville Road when accident
occurred
doesn’t remember, but thinks sedan hit the
truck
drivers had a shouting match
truck struck another car on Plymouth
Avenue before stopping
heard volunteer EMT say the truck driver
was intoxicated
Andrew Mavros:
saw red truck had rear-ended white sedan
near Washington Avenue and Plymouth
Avenue
was almost hit by truck as the driver
attempted to flee the scene
saw truck hit another car
went to go help truck driver
After reading the passage and taking notes, you can evaluate each of the answer options to determine which is
true. First, we consider option a.” You should refer to your notes and find that both Hailey House and Travis Quin
mentioned intoxication. However, Hailey only guessed that the truck driver was intoxicated. Further, Travis only
mentions that he heard a volunteer EMT (who we know to be Hailey, based on her account) say that the truck
driver was intoxicated. Therefore, we cannot logically conclude that the truck driver was indeed intoxicated.
Option “a” is incorrect.
Next, we should evaluate option “b.” According to our notes, three of the individuals made statements regarding
who was at fault for the accident. Travis states that he doesn’t remember who was at fault, but he thinks the
sedan hit the truck. However, the two other witnesses who saw the aftermath of the initial accident, Marvis and
Andrew, both say they saw the red truck had rear-ended the white sedan. Based on this information, we cannot
conclude that the initial accident was caused by the white sedan; it is more likely that it was caused by the red
truck. Option “b” is incorrect.
Cognitive Ability Measure
New York State Police Trooper Examination Study Guide 19
Copyright © 2022, IOS, Inc.
Now consider option “c.” Both Marvis and Hailey mentioned an off-duty trooper. Marvis stated that the truck
driver almost hit an off-duty trooper. Hailey stated that she saw an off-duty trooper assisting a man near a red
truck at the scene. In combination with Andrew’s statements that he was the person who was almost hit by the
truck and that he went to help the truck driver after he hit a second car, we can conclude that Andrew Mavros is
the off-duty trooper that both Marvis and Hailey saw. Therefore, option “c” is the correct answer.
Although option “c” is correct, we should still examine option “d” to eliminate it. Option “d” states that the truck
driver attempted to hit a pedestrian. Based on the witness accounts, there is no information that confirms this.
We are not provided with any information as to the truck driver’s motives, so option “d” cannot be correct. Option
“c” is the best answer.
Correct answer: C
Inductive Reasoning Tips
Given the volume of information that you must synthesize, it is necessary to make some form of notes or diagram
to generate a reasonable understanding of the situation that is described. Commonly, candidates will error on
these types of questions because they are rushing and fail to generate a clear picture of the situation.
Ensure that you do not draw conclusions based on a single individual’s account. You should synthesize the
information provided by all individuals in order to draw conclusions and answer the questions that are asked. Do
not rely on assumptions made by witnesses; you should only rely on the facts that are presented.
All the questions on the New York State Police Trooper Examination that assess inductive reasoning will involve
the synthesis of witness or bystander accounts of various situations. To improve your inductive reasoning and
ultimately perform at your best on the NYSP Trooper Examination, you should refer to Additional Resources for
Improving Your Cognitive Abilities at the end of the Cognitive Ability Measure section.
Cognitive Ability Measure
New York State Police Trooper Examination Study Guide 20
Copyright © 2022, IOS, Inc.
PATTERN RECOGNITION
Pattern recognition is the ability to determine the underlying rule, process, or transformation that is present
within a set of materials. It involves identifying a pictorial or visual pattern when parts of the pattern are
presented.
Troopers exercise this ability when analyzing a wide range of cases to look for patterns or similar crimes. Troopers
may look for patterns in factors such as victim descriptions, methods of entry, types of goods taken, etc.
The following question will allow you to assess your pattern recognition ability, identify any areas of weakness,
and learn the proper method for solving this type of question.
Sample question 8
For Sample Question 8, please select the answer that completes the pattern.
8.
a.
b.
c.
d.
Discussion
This question measures pattern recognition, or your ability to identify a visual pattern when parts of the pattern
are presented. In this case, you need to consider the presented pattern and choose the shape that fits into the
blank space containing the question mark.
Begin by determining the direction of the pattern in the presented image (does the pattern move horizontally or
vertically across the image?). In this image, it appears that the pattern moves horizontally, as each row contains
Cognitive Ability Measure
New York State Police Trooper Examination Study Guide 21
Copyright © 2022, IOS, Inc.
its own set of similar blocks. Next, attempt to identify the change from one item in the sequence to the next. It
could be helpful to make a note next to the image indicating the type of transformation that has occurred. In the
pattern above, start with the top row. From the left image to the center image, the tile has been flipped vertically.
From the center image to the right image, the tile has been flipped horizontally.
To confirm that this is the pattern that exists throughout the image, next you should examine the second row.
From the left image to the center image, the tile has been flipped vertically. From the center image to the right
image, the tile has been flipped horizontally. This is the same pattern as the first row, and confirms that this is the
pattern that should be replicated in the third row.
Note that the center image in the third row contains a question mark. This means that you should choose the
answer that would fit into this center tile. Based on the previous two rows, we know that the center image should
be a vertical flip of the left image. Now look at the shape before the blank shape. In your mind, apply the
transformation you noted to this shape. Visualize what the image should look like, and then select it from the
answer options.
Option “b” is the answer option that presents a vertical flip of the left image, and therefore correctly completes
the presented pattern. Option “b” is correct.
Correct answer: B
Pattern Recognition Tips
When approaching a pattern recognition item, you should start small. Identify the transformation that occurs
from the first shape in the pattern to the next. After you have determined the transformation that has occurred,
identify the transformation that occurs for the next set of shapes. Some possible patterns to look for include:
rotation, vertical or horizontal translation, changes in shape, and changes in size.
For some pattern recognition items, like Sample Question 8, you will find that the pattern moves horizontally from
one shape to the next. For other items, the pattern will move vertically through the image. Before trying to identify
the correct answer, first make sure you have correctly identified the pattern within the image.
A common mistake made by candidates is to not pay close enough attention to the differences between answer
options. It can be very overwhelming to solely skim each option, especially when all of the options will be very
similar. While skimming the answer options will help you to save time and eliminate obviously incorrect options,
you may miss small differences. When the wrong strategy is used, candidates often fail to notice the small
differences that identify which answer options are incorrect.
To improve your pattern recognition and ultimately perform at your best on the NYSP Trooper Examination, you
should refer to Additional Resources for Improving Your Cognitive Abilities at the end of the Cognitive Ability
Measure section.
Cognitive Ability Measure
New York State Police Trooper Examination Study Guide 22
Copyright © 2022, IOS, Inc.
GRAMMAR
Grammatical ability reflects an individual’s knowledge of writing with respect to word usage, spelling, and
punctuation.
Troopers engage in written communication in a multitude of ways; therefore, troopers must be able to
communicate intelligently and professionally via the written word. It is necessary to understand how to spell
common words, properly use and pair parts of language (e.g., nouns, verbs, articles, etc.), punctuate sentences
properly, and compose meaningful sentences.
The following question will allow you to assess your grammatical ability, identify any areas of weakness, and learn
the proper method for solving this type of question.
Sample question 9
For Sample Question 9, choose the answer option that correctly completes the sentence.
9.
As you become more tenured, you will _______ more time off.
a.
aquire
b.
adquire
c.
acquire
d.
acquier
Discussion
This question measures your ability to identify proper spelling. To the extent that you understand how to spell
appropriately, you should be able to recognize which answer option correctly completes the sentence provided.
In order to answer this question, you should first read the sentence and answer options and see if the correct
answer noticeably stands out to you. If so, try inserting what you believe to be the correct spelling into the
sentence and re-reading it to ensure that it is accurate. If you cannot immediately identify the correct answer,
carefully read through the answer options to identify which ones contain errors. Move through each option using
this method. Make sure you can come up with a spelling rule that is clearly broken in each instance. Through this
process of elimination, you should end up with only the correct answer remaining.
In this question, the only correct answer is “c. Specifically, the word acquireis spelled correctly. Each of the
other answer options presents misspellings of the word “acquire.” By utilizing your knowledge of basic spelling,
you can conclude that the correct answer is option “c.
Correct answer: C
Grammar Tips
All of the grammar items on the examination will ask you to identify the answer option that correctly completes
a sentence. This type of question will require you to be familiar with common grammar, spelling, punctuation, and
syntax rules. There are many rules that govern the use of the English language. It would be wise to consult a text
that teaches high school-level (8
th
to 10
th
grade) grammar and style rules, or a writer’s style manual. Please note
that the examination will only ask questions about conventional and commonly-accepted practices. Style manuals
often disagree on lesser grammar and style rules. Also, any misspelled words for which questions are asked will
be commonly used words that the majority of individuals should be capable of spelling.
Cognitive Ability Measure
New York State Police Trooper Examination Study Guide 23
Copyright © 2022, IOS, Inc.
Following is a brief summary of common and important grammar and syntax rules. This list is not intended to be
exhaustive, but rather to communicate common rules and errors with which you should be familiar.
Use of Apostrophes and Quotation Marks
If you are trying to communicate the possessive form of a singular noun, generally you add “ ’s” to the word.
Incorrect: The childs bike was stolen from her front yard.
Correct: The child’s bike was stolen from her front yard.
If the noun is plural, generally, you add the apostrophe after the “s.”
Incorrect: The twins’s parents allowed both of them to attend the party.
Correct: The twins’ parents allowed both of them to attend the party.
The possessive form of it” does not contain an apostrophe (the apostrophe is reserved for the contraction of “it
is”).
Incorrect: Sandra had to replace her radio after it’s speaker stopped working.
Correct: Sandra had to replace her radio after its speaker stopped working.
Possessive pronouns (e.g., hers, yours) do not use apostrophes.
Incorrect: Meghan is known for her landscaping skills. Her’s is the biggest garden on the block!
Correct: Meghan is known for her landscaping skills. Hers is the biggest garden on the block!
When contracting words, the apostrophe is intended to take the place of the missing letter(s) (e.g., “it’s” for “it is,”
wouldn’t” for “would not”).
Incorrect: Josh could’nt go to work today because he was feeling sick.
Correct: Josh couldn’t go to work today because he was feeling sick.
Generally, a quote contained within a sentence is proceeded by a comma outside the quotation marks and/or is
followed by a comma inside the quotation marks.
Incorrect: Mark said “I really hate when the air conditioner stops working.
Correct: Mark said, “I really hate when the air conditioner stops working.
OR
Incorrect: “I really hate when the air conditioner stops working” Mark said.
Correct: “I really hate when the air conditioner stops working,” Mark said.
Subject-Verb Agreement and Verb Tense Agreement
The number (singular or plural) of the subject dictates the number of the verb.
Incorrect: The girls runs to school.
Correct: The girls run to school.
OR
Incorrect: The dog chase the bus.
Correct: The dog chases the bus.
When the subject is a pronoun, the pronoun number and verb number must agree.
Incorrect: People stare at him as he patrol the neighborhood.
Correct: People stare at him as he patrols the neighborhood.
He/she/his/her coincide with singular nouns and they/their coincides with plural nouns.
Incorrect: Trooper Summers took their lunch break at noon.
Correct: Trooper Summers took her lunch break at noon.
OR
Incorrect: Trooper Ortega and Sergeant Byrd parked his squad car in the shade.
Correct: Trooper Ortega and Sergeant Byrd parked their squad car in the shade.
Cognitive Ability Measure
New York State Police Trooper Examination Study Guide 24
Copyright © 2022, IOS, Inc.
The verb tense (present, past, future) should remain consistent within a summary or description.
Incorrect: Austin left his house to go to work. He gets in his car and started driving down the road. As he
was stopped at an intersection, he heard brakes squealing and sees a white truck run the red light. It crashed into
a blue sedan and flees the scene.
Correct: Austin left his house to go to work. He got in his car and started driving down the road. As he was
stopped at an intersection, he heard brakes squealing and saw a white truck run the red light. It crashed into a
blue sedan and fled the scene.
Commonly Misused Words
Numerous words are commonly confused or misused including, whether/weather, affect/effect, then/than,
their/there/they’re and to/too.
Incorrect: Before approaching the suspect, Trooper Butler debated weather or not he should call for backup.
Correct: Before approaching the suspect, Trooper Butler debated whether or not he should call for backup.
OR
Incorrect: Sergeant Cohen was called too the scene of a burglary.
Correct: Sergeant Cohen was called to the scene of a burglary.
OR
Incorrect: Troopers Griffin and Hubbard returned from they’re lunch break to find a vandalized squad car.
Correct: Troopers Griffin and Hubbard returned from their lunch break to find a vandalized squad car.
To improve your grammar and ultimately perform at your best on the NYSP Trooper Examination, you should also
refer to Additional Resources for Improving Your Cognitive Abilities at the end of the Cognitive Ability Measure
section.
Cognitive Ability Measure
New York State Police Trooper Examination Study Guide 25
Copyright © 2022, IOS, Inc.
VISUALIZATION
Visualization is the ability to imagine how something will look when it is moved around or when its parts are
moved or rearranged. It requires the forming of mental images of how people would look after certain changes,
such as physical alterations. One has to predict how a person will appear after the changes are made.
Often, troopers are presented with BOLOs (be on the lookout) containing pictures and descriptions of suspects.
Since suspects may change their appearance by coloring their hair, changing their hair style, wearing glasses or
hats, etc., troopers will need the ability to recognize a face despite minor changes that are made to one’s
appearance.
The following question will allow you to assess your visualization ability, identify any areas of weakness, and learn
the proper method for solving this type of question.
Sample question 10
For the following question, an image of a face is presented. You will be asked to select an identical face from among
a series of four faces to which various manipulations have been made.
10.
This man removed his eyeglasses. Which of the images below is the same man,
but without his eyeglasses?
a.
b.
c.
d.
Discussion
This question measures visualization, or your ability to identify or detect a known facial image that has been
hidden in other distracting material such as similar facial images and altered features like a changed hair style,
facial expression, or the addition or removal of glasses.
The question provides information regarding the change that has taken place to the target image. In this case, an
image of a man is presented along with the statement that he has removed his eyeglasses. You are to identify the
same man presented in the target image among the multiple distracting images. Each distracting image is designed
to mirror certain aspects of the target image, though only one is an exact duplicate of the target image.
Cognitive Ability Measure
New York State Police Trooper Examination Study Guide 26
Copyright © 2022, IOS, Inc.
Faces are like any other object; they are made up of different sized shapes i.e., facial features. As such, the
difference between two faces can be broken down into a comparison of shapes. In order to correctly answer this
question, you will need to break down and compare more finite shapes found within the faces. You should start
your comparison by comparing facial features across the faces. For example, compare the eyes, nose, lips, jawline,
brow, bridge, cheeks, and head shape. A good strategy is to start with larger features and work your way through
smaller features. Of note is that the skin tone, eye color, or other colorations are not intentionally manipulated.
Differences noted here may be caused by shadowing effects of the image due to a different facial angle or hair
feature. You should identify differences between the faces only by comparing facial features.
Once you begin comparing more finite facial features, differences between the various faces will begin to emerge.
Starting with a larger feature, such as the head shape, compare the target image to all options choices. When you
isolate the head shape, it should reveal that option “b” has a much broader (wider) face. This larger feature plays
out further when you consider the related smaller features. For example, option “b” has much fuller cheeks than
the target, and a wider jaw line. As such, option “b” can be eliminated from further consideration. Looking at the
head shape also reveals that option “d” has a much shorter face (rounder vs. more oval-shaped), with a less-
defined chin than the target image. Option “d” can thus be eliminated.
Options “a” and cboth have a more similar face shape to the target, but option “a” has a narrower overall head
shape with gaunter (thinner) cheeks and a narrower jaw line. Such observations should be noted; however, if you
still have some uncertainty regarding which option of the remaining options is the exact match to the target, then
you should continue to examine other facial features. Next, we examine the eyes/eyebrows and nose features of
options “a” and “c” in comparison to the target. This comparison reveals that the brow ridge of option “a” is less
pronounced than that of the target image. Additionally, option “a” has a slimmer nose than the target’s.
Therefore, option “a” can be eliminated, leaving only option “c.” Option “c” is the correct answer; it is the same
face as the target image.
Correct answer: C
Visualization Tips
The exam will contain questions that are very similar to the one presented above. In each case, a target image of
a face will be presented along with a question identifying changes that have occurred to the target image. You will
need to identify the correct answer option that contains the same face as the target image.
A common mistake made by candidates is to not break down the image into smaller pieces. It can be
overwhelming to try to take in all the information at once, especially when two faces are very similar. The strategy
for any question of this dimension type is to deal with smaller subsets of information. When the wrong strategy
is taken, candidates often fail to notice small differences that identify which option choices are incorrect. Instead
candidates will focus on what is the same rather than what is different.
To improve your visualization and ultimately perform at your best on the NYSP Trooper Examination, you should
refer to Additional Resources for Improving Your Cognitive Abilities at the end of the Cognitive Ability Measure
section.
Cognitive Ability Measure
New York State Police Trooper Examination Study Guide 27
Copyright © 2022, IOS, Inc.
SPATIAL ORIENTATION
Spatial orientation is the ability to understand how to navigate within spaces or how to get from one point to
another. This ability involves imagining yourself at a certain location and orientating yourself to the direction you
would move to get to another location. It also involves knowing your location in relation to the environment or
knowing where other objects are in relation to you.
Troopers require this ability to travel from their current location to an emergency scene and also to navigate in
and out of a building under possible duress (e.g., raids). Troopers also use this ability when they have to reach a
destination in a community. They have to be able to visualize where they are in relation to their destination.
The following question will allow you to assess your spatial orientation ability, identify any areas of weakness, and
learn the proper method for solving this type of question.
Sample question 11
Please use the map below to answer Sample Question 11.
N
Cognitive Ability Measure
New York State Police Trooper Examination Study Guide 28
Copyright © 2022, IOS, Inc.
11.
You are at the intersection of Southwest 9th Street and Southwest High Avenue. You drive the
following route: south on Southwest High Avenue, west on Southwest 10th Avenue, north on
Southwest Hedgewood Avenue, east on Southwest 8th Avenue, north on Southwest Randolph
Avenue, and then stop at the first intersection. Which of the following intersections is closest to your
new position?
a.
Southwest 9th Street and Southwest Randolph Avenue
b.
Southwest Randolph Avenue and Southwest 6th Avenue
c.
Southwest Medford Avenue and Southwest 8th Avenue
d.
Southwest 7th Street and Southwest Webster Avenue
Discussion
This question measures spatial orientation, or your ability to understand how to navigate within spaces or how to
get from one point to another.
In order to answer this question, you will need to familiarize yourself with the map. Notice that there is a
directional compass on the upper right-hand side of the map. On the following page, you will find a map with the
route from the stem traced out and with the intersections from the answer options labeled.
In referencing the question, you should imagine you are currently at the corner of Southwest 9th Street and
Southwest High Avenue (yellow asterisk on map on the following page). You should place some sort of indicator
at this intersection on the map, so you know where you will begin. Additionally, you should place an indicator at
each of the intersections from the answer options (blue dots labeled “a”-“d” on the map on the following page),
so you know the points that you must compare to determine which is closest to where you end up.
Cognitive Ability Measure
New York State Police Trooper Examination Study Guide 29
Copyright © 2022, IOS, Inc.
The most effective method for approaching this type of question is to first trace the path that is indicated in the
item’s stem. This path is traced in yellow on the map. This route will take you from the beginning point at
Southwest 9
th
Street and Southwest High Avenue to the end point at Southwest 7
th
Street and Southwest Randolph
Avenue. You should place an indicator at this end point on the map (yellow “x” above).
You should now look at the end point (yellow “x”) in relation to each of the intersections you marked on the map.
In doing so, you will see that option “d,” Southwest 7
th
Street and Southwest Webster Avenue, is the closest
intersection to the end point. Therefore, option “d” is the correct answer.
Correct answer: D
Spatial Orientation Tips
The most common mistakes on spatial orientation questions have to do with improperly orienting yourself. First,
you must understand where north, south, east and west are based on the map you are viewing. You also need to
N
Cognitive Ability Measure
New York State Police Trooper Examination Study Guide 30
Copyright © 2022, IOS, Inc.
be sure that you consider the perspective of an individual who is inside” the map. This will allow you to make
correct judgments about whether a turn is left or right, east or west, or north or south. Remember to carefully
review the map before reading the questions as this will help you better relate to the question.
To practice your spatial orientation ability, you can print out a map of your city. Choose a starting point and an
ending point, and practice writing out directions to get from one point to the next. Doing this will allow you to
familiarize yourself with the cardinal directions, and you will gain practice mapping out routes of travel.
To improve your spatial orientation and ultimately perform at your best on the NYSP Trooper Examination, you
should also refer to Additional Resources for Improving Your Cognitive Abilities at the end of the Cognitive Ability
Measure section.
Cognitive Ability Measure
New York State Police Trooper Examination Study Guide 31
Copyright © 2022, IOS, Inc.
FLEXIBILITY OF CLOSURE
Flexibility of closure is the ability to identify or detect a known pattern (such as a figure, object, word, or sound)
that is hidden in other distracting material.
Troopers use flexibility of closure to pick out a disguised pattern, person, or object that is somehow hidden within
a group of other objects. Troopers rely on their flexibility of closure ability to pick out a particular suspect’s face
from a crowd of people. Troopers also use flexibility of closure when looking for a particular license plate amongst
heavy traffic.
The following questions will allow you to assess your flexibility of closure ability, identify any areas of weakness,
and learn the proper method for solving these types of questions.
Sample question 12
Please use the image below to answer Sample Question 12.
12.
Which of the following license plate numbers appears in the above image?
a.
RNA 2341
b.
GSM 0235
c.
MLC 5733
d.
GSM 0521
Cognitive Ability Measure
New York State Police Trooper Examination Study Guide 32
Copyright © 2022, IOS, Inc.
Discussion
The method you should use to answer this type of question is to systematically scan the presented image for each
answer option. Read each license plate number in the image while searching for option “a.” Then do the same for
options “b,” “c,” and “d.” The correct answer is “b.” The license plate number “GSM 0235” is found on the third
license plate in the second row of the image. None of the other license plate numbers are included in the image.
Correct answer: B
Sample question 13
Please use the image below to answer Sample Question 13.
13.
How many times does the following face appear?
a.
One time
b.
Two times
c.
Three times
d.
Four times
Cognitive Ability Measure
New York State Police Trooper Examination Study Guide 33
Copyright © 2022, IOS, Inc.
Discussion
For this type of flexibility of closure item, you should scan the presented image to determine how many times the
given target image appears. It may be helpful to break the task down by examining each of the four quadrants of
the image separately.
Start by looking in the top left quadrant. In this quadrant, the target image appears two times (the second image
in the top row and the third image in the bottom row). The target image does not appear in the top right quadrant.
In the bottom left quadrant, the image appears twice (the fourth image in the top row and the fourth image in
the bottom row). Finally, the target image does not appear in the bottom right quadrant.
Adding up the two instances in the top left quadrant and the two instances in the bottom left quadrant, we get a
total of four instances of the target image. Therefore, option “d” is the correct answer.
Correct answer: D
Flexibility of Closure Tips
The most common mistakes on flexibility of closure items that are similar to Sample Question 12 occur because
candidates are rushing through the items. When candidates rush, they often fail to notice small differences that
identify which option choices are incorrect. When you are presented with license plate numbers in the answer
options or question stems, make sure you read the entire number. You should also ensure that you read the entire
plate numbers that are found within the image. Some numbers might start or end the same way but have slight
differences in the middle. If you are quickly skimming over the image, you might fail to recognize these differences.
The most common mistake on the flexibility of closure items similar to Sample Question 13 is failure to count all
of the instances of the given target within the presented image. Searching the four quadrants of the image
separately will break the task down into simpler components and will allow you to focus on smaller portions when
searching. In doing so, you are less likely to skip over an instance of the target image within the picture.
To improve your flexibility of closure and ultimately perform at your best on the NYSP Trooper Examination, you
should refer to Additional Resources for Improving Your Cognitive Abilities at the end of the Cognitive Ability
Measure section.
MEMORIZATION
Memorization is the ability to remember information such as words, numbers, pictures, and procedures, and to
recall specific information upon request.
Troopers use this ability to remember the important information presented in statutes, legal bulletins, shift
briefings and BOLOs and also to remember new names, faces, codes, telephone numbers, license plate numbers,
geographic locations, documents, and long lists, which can be in the form of words, numbers, pictures, and
procedures.
For the memorization questions on the exam, you will be presented with three photographs. You will have six
minutes to study all three photographs. After the six minutes have elapsed, you will answer several questions
about the pictures you saw. You will not be allowed to look back at the pictures while you are answering the
questions.
The following question will allow you to assess your memorization ability, identify any areas of weakness, and
learn the proper method for solving this type of question.
Cognitive Ability Measure
New York State Police Trooper Examination Study Guide 34
Copyright © 2022, IOS, Inc.
Sample question 14
Spend two minutes observing the picture below and memorize any and all possible information provided in the
picture. Then, you should answer the practice question regarding the information in the picture you just observed.
In order to practice for the actual exam, we recommend that you do not look back at the picture while you are
answering the practice question.
Look at the following picture for two minutes and memorize as many details as possible. Rely on your memory of
this image to answer the question that follows. Please use the image below to answer Sample Question 14.
Cognitive Ability Measure
New York State Police Trooper Examination Study Guide 35
Copyright © 2022, IOS, Inc.
14.
What color are the officers’ shirts in the photo?
a.
White
b.
Black
c.
Green
d.
Blue
Discussion
The correct answer to Sample Question 14 is “d” because the officers’ shirts are blue.
Correct answer: D
Memorization Tips
Each memorization item will require you to recall specific details from a presented image. Practice on your own
by studying a picture for no more than two minutes. Then try to describe everything you observed in the picture
or have someone else ask you questions about the picture. Try to answer the questions without looking at the
picture, because you will not be able to look back at the picture on the actual exam. It would be best to use a
picture that you have never seen before, such as a picture from a newspaper or magazine.
To improve your memorization and ultimately perform at your best on the NYSP Trooper Examination, you should
also refer to the following section, Additional Resources for Improving Your Cognitive Abilities.
Cognitive Ability Measure
New York State Police Trooper Examination Study Guide 36
Copyright © 2022, IOS, Inc.
ADDITIONAL RESOURCES FOR IMPROVING YOUR COGNITIVE
ABILITIES
Your performance on the cognitive ability section of the examination can be improved by practicing the specific
cognitive abilities that are measured by the exam. For more practice with these abilities, you should consider the
following resources:
NYSP TROOPER PRACTICE TEST. The Practice Test at the end of this guide contains a full 65-item
cognitive ability test that is similar to the actual NYSP Trooper Examination. For the best chance at improving
your cognitive abilities and ultimately performing at your best on the NYSP Trooper Examination, you should
complete the Practice Test. Following the test are an answer key and a score interpretation guide so that you
can check your answers and determine where you should focus your study efforts.
GRAMMAR, SPELLING, AND VOCABULARY WORKBOOKS. For additional resources to
improve your grammar and vocabulary, consider referencing a study aid that focuses on high-school level (8
th
to 10
th
grade) grammar, spelling, and/or vocabulary. Appropriate study aids for the NYSP Trooper Examination
would be tailored to individuals studying for high school or for the GED test.
STUDY AIDS FOR IOS’ FLAGSHIP ENTRY-LEVEL LAW ENFORCEMENT
EXAMINATIONS. IOS publishes several entry-level law enforcement examinations which measure
cognitive abilities similar to those measured by the NYSP Trooper Examination. While the formats of some of
the questions found on these examinations differ from the formats of those on the NYSP Trooper Examination,
the underlying cognitive abilities measured by the examinations are similar. Reviewing study materials for the
following examinations may prove helpful if you need additional practice:
o NCJOSI
2
. Preparation materials for the NCJOSI
2
will help you hone the following cognitive abilities:
deductive reasoning, flexibility of closure, inductive reasoning, spatial orientation, and visualization.
NCJOSI
2
study aids also help with verbal comprehension (similar to the reading items on the NYSP Trooper
Examination), information gathering (similar to ordering of information), and verbal expression (similar to
the writing and grammar sections on the NYSP Trooper Examination). NCJOSI
2
study aids do not contain
information pertaining to memorization, vocabulary, or pattern recognition items.
o LST. Preparation materials for the LST will help you with the following cognitive abilities: deductive
reasoning, flexibility of closure/visualization, inductive reasoning, information ordering, and spatial
orientation. LST study aids also help with written comprehension (similar to the reading items on the NYSP
Trooper Examination) and written expression (similar to writing and grammar items). LST study aids do
not contain information pertaining to memorization, vocabulary, or pattern recognition items.
o You can find these study aids on IOS’ website at https://iosolutions.com/ios-web-store. Use the product
navigation bar to find study materials for the exams mentioned here, or use the product search bar. If you
have questions regarding these products, please contact IOS at 888-784-1290 or info@iosolutions.com.
Personality Attributes Measure
New York State Police Trooper Examination Study Guide 37
Copyright © 2022, IOS, Inc.
The personality attributes section of the examination contains two parts. The first part will ask you to determine
your level of agreement with a variety of statements using the five-point agreement scale shown below.
A
B
C
D
E
Strongly
Disagree
Disagree
Neutral
Agree
Strongly
Agree
When answering questions in this section, you should briefly consider the statement that is presented and then
provide a response that corresponds with your initial reaction regarding your level of agreement or disagreement.
Your first reaction is usually the most candid and accurate response.
The second part of the personality attributes section will present you with two statements and ask you to select
the statement you MOST agree with. Again, you should go with your initial reaction to ensure that your responses
are candid and accurate.
SAMPLE QUESTIONS
Part 1 Sample Questions
This section contains a series of statements that you must evaluate based on your personal opinion. Read each
statement and then provide a response based on the agreement scale above.
1. I am sometimes nervous around friends whom I have not seen for a long time.
2. I enjoy trying to make others feel comfortable in new situations.
3. When I am uncertain of how to do a task, I will usually ask for clarity.
Discussion
You may agree with item 1. You would thus assign answer option Dto item 1. You may
disagree with item 2. You would thus assign answer option “B” to item 2. Finally, you
may strongly disagree with item 3. You would thus assign answer option “A” to item 3.
Part 2 Sample Questions
For this section, please choose the response option that most closely identifies with your beliefs or attitudes. If you
do not exactly identify with one option or if you are unsure, rely on your first impulse.
1. A. I am more assertive than most of my friends.
B. Others would describe me as timid.
2. A. When I get home from work or school, I usually feel stressed.
B. I do not bring my feelings about work or school home with me.
Discussion
Assume that for item 1, you agree with statement A, “I am more assertive than most of my
friends. You would thus select option A” for item 1. Next, assume that for item 2, you
agree with statement B, “I do not bring my feelings about work or school home with me.”
You would thus select option “B” for item 2.
PERSONALITY ATTRIBUTES SECTION TIPS
Respond to each statement candidly according to how much you agree or disagree with it. Your response should
be an indication of how you feel about the statement that is presented. You should respond honestly to each
1. A B C D E
2.
A B C D E
3. A B C D E
4. A B
5. A B
Pe
rs
on
ali
ty
Att
rib
ute
s
Me
as
ure
Personality Attributes Measure
New York State Police Trooper Examination Study Guide 38
Copyright © 2022, IOS, Inc.
statement. A good guideline for answering these types of items is to put down the first response that feels right.
Overanalyzing the statement or looking for hidden meaning can often cause you to misunderstand the statement
and respond in a manner that does not reflect your true behavioral attributes.
Responding Honestly
This section of the examination has been developed so that responding in a dishonest manner to “look good” or
to “game” the test will result in lower scores. For example, in Part 1 of this section, strategies such as using only
extreme parts of the scale (using only “1s” and “5s”) or other “patterns” are flagged by the examination, and will
result in lower scores. We caution all test takers to avoid any such strategies. Any information, resources or
testimonials that you may encounter suggesting that these types of strategies are successful are ill-informed.
Following this ill-informed advice will likely result in the lowering of your score, impacting your chance of passing
the New York State Police Trooper Examination. Responding honestly according to your personal thoughts,
feelings, or opinions is the best strategy.
General Test-Taking Tips
New York State Police Trooper Examination Study Guide 39
Copyright © 2022, IOS, Inc.
TIPS FOR THE DAY OF THE EXAMINATION
Your physical well-being. Get a good night’s sleep before the exam. Try to ensure that you are in the best possible
condition both physically and mentally on the day of the exam.
Arrive early. Make all necessary arrangements to ensure that you arrive early at the test site. You may need to make
advance arrangements for such things as childcare and a ride to the test site. It would also be wise to anticipate traffic
delays in traveling to the test site and to allocate extra travel time to ensure an early arrival even with delays. Give
yourself ample time to settle in at the test site. If you arrive late, you may not be admitted to the test.
Confidence. The more confident you are in your abilities, the more likely you are to do well on the exam. Try to stay
focused so that you don’t end up reading sentences and answer choices over and over again. You want to take control
of the exam by feeling confident in your skills. Try to relax but concentrate on the exam you are about to take.
Attitude. This exam is an opportunity for you to show your skills and abilities, and a positive attitude can have an
impact on increasing your test score. There are a few ways to fine-tune your attitude about taking this exam:
o Look at this exam as a challenge but try not to get “stressed out” by thinking about it too much.
o Remember that by using this study guide and practicing your strategies, you can be more prepared for the
exam.
GENERAL STRATEGIES FOR TAKING THE EXAMINATION
Listen carefully. The exam administrator will provide you with complete instructions for taking the exam. Be sure that
you pay close attention to all test instructions. It is extremely important that you completely understand the directions
before the exam begins.
Ask questions. If there are instructions that you do not understand or if something is not clear, please feel free to ask
the exam administrator any questions that you may have before the exam begins.
Note start and end times. Just before the test administrator begins the test, make a note of the time that the test will
actually begin and the time that the test will end.
Keep track of time. It is important that you keep track of your time so that you are able to pace yourself throughout
the exam. You do not want to run out of time and have unanswered questions. Since the score on the cognitive
portion of your exam will be based on the total number of correct answers, you will want to work both quickly and
efficiently. It is also important that you keep track of time so that you are able to allot a few minutes to guess on
unanswered questions at the end of the exam.
Remain calm. It is important that you remain calm throughout this exam so that you are able to work most effectively.
You can’t allow yourself to become nervous or confused by a few difficult or challenging questions. There are going
to be times when you are not going to know the answer, and neither will other individuals taking the exam. If you
believe that you have not done well on any particular type of question, try to remain calm and focus on doing your
best for the rest of the exam.
Read each question carefully. Be sure that you understand what the question requires of you.
Read for meaning. Don’t get stuck on words or sentences you do not understand. You may be able to get the main
idea from a sentence or paragraph without understanding the individual words or the individual sentences.
Try to answer the question correctly before you look at the answers given. If you know the answer, compare it to
the available choices and pick the choice closest in meaning to the answer you have in mind.
Answer easy questions first. It is important that you answer the easy questions first on this exam. By doing so, you
will afford yourself more time to spend on the difficult questions if necessary. Seek out questions you can answer. If
there are particular kinds of questions that you feel most comfortable with, answer those questions first. For example,
if you are particularly good at reading comprehension, find those items first and complete them.
G
e
n
e
r
a
l
T
e
s
t
-
T
a
k
i
n
g
T
i
p
s
General Test-Taking Tips
New York State Police Trooper Examination Study Guide 40
Copyright © 2022, IOS, Inc.
Don’t waste time on questions you can’t answer. If you believe that you are not able to answer a question, then skip
it. It is better to skip a difficult question than to spend a lot of time on it and not be able to complete other exam
questions.
Mark your exam booklet. Mark in your exam booklet those questions that you have skipped so that you are able to
come back to them later in the exam. Marking questions that you have skipped in your test booklet will help to ensure
that you have answered all of the questions by the time the examination period has ended.
Use a process of elimination. If a situation presents itself where you are unable to answer a question, use a process
of elimination to narrow your options. First, eliminate any answers that are clearly wrong. Then determine how you
feel about the remaining choices. If you believe that some answers are more correct than others, indicate that on
your exam booklet. Later in the exam, you may have an opportunity to come back to these questions and answer
them without having to read the entire question again.
Guessing. It is important that you try to get as many points as possible on the examination. Points are given for correct
answers only. Because there is no penalty (i.e., points deducted) for guessing or incorrect answers on this exam, you
should try to answer every question. By leaving yourself enough time at the end of the test to fill in unanswered
questions using your “best guess,” you will increase the possibility of getting a few of them correct.
Extra time. If you finish the exam before the time period is over, review your answers and make any changes that are
necessary. Also make sure that you have marked your answers on the answer sheet correctly.
RESPONDING TO QUESTIONS
For the cognitive ability section of the test, your score will be based on the total number of questions you answer correctly.
No additional points are deducted for marking incorrect responses; therefore, it is in your best interest to answer all
questions on the exam. Your failure to answer all questions could have a detrimental effect on your test score. It is
advisable to make an educated guess on a test question rather than leaving the question blank.
If you are having trouble identifying the correct answer for a question, it may be beneficial to skip the question and come
back to it as time allows. This will keep you from wasting valuable time on questions that you are less likely to respond to
accurately.
You will record your answers on a computer-read answer sheet. This type of answer sheet is highly sensitive. Be careful
not to make errant marks on the answer sheet or use the answer sheet as scratch paper. Fill in all answer spaces according
to the instructions provided prior to the administration of the test. Ensure that your answers are legible and are clearly
and darkly marked. Also, frequently check to make sure that the number of the question that you are answering
corresponds to the number on the answer sheet.
Quick Tips for Answering Multiple-Choice Questions
Read the entire question carefully and try to answer it without referring to the answer choices.
Look for any key words in the question that may help you select the correct answer from among the choices. Some
common key words are: many, most, least, less, more, good, best, advantage, disadvantage, first, last, never, always,
any, not, except, false, and true.
Always read all of the possible answer alternatives carefully before jumping to the conclusion that a particular one
must be the best.
Mark only one answer for each question. If you mark more than one answer, that question will be considered
incorrect, and you will not receive credit for your answer.
Use the process of elimination if the correct answer does not immediately occur to you. Eliminate obviously wrong
answers and narrow your choice to the ones that directly answer the question. Then select the answer that most
exactly answers the question.
General Test-Taking Tips
New York State Police Trooper Examination Study Guide 41
Copyright © 2022, IOS, Inc.
Do not be influenced by the length of the answer choices. The longest answer is not necessarily correct.
Do not select an answer choice just because it includes technical language. Answer choices using technical terminology
may be included in order to see whether you know the difference between what "looks right" and what "is right."
Determine the best answer using only the information supplied in the question, without making unwarranted
assumptions. The correct answer is the one that works best for the situation described.
STRATEGIES FOR ANALYZING AND AVOIDING ERRORS
Four common causes for choosing incorrect responses are presented to you below, along with suggestions for avoiding
these errors. Reading through this section carefully can help you identify and correct any major weaknesses in your test-
taking behavior.
1) Answer sheet marking errors and guessing errors.
Reason: You may have missed a question because you skipped it and failed to come back to it later.
Suggestion: Be sure you mark skipped questions in your exam booklet and remember to go back to them before the end
of the exam period.
Reason: You may have lost track of the time and been unaware that the exam period was about to end before you could
mark any remaining unanswered questions.
Suggestion: Be sure to check your watch often (time goes by quickly during an exam) so that you can keep track of how
much time you have left. Leave five minutes at the end of the exam so that you are able to go back to any unanswered
questions and complete them.
Reason: You may have been forced to make guesses for questions at the end of the exam because you spent too much
time working on difficult questions rather than skipping them and saving them for later.
Suggestion: If you did skip difficult questions, it may be because you failed to narrow down the number of alternatives in
the question.
2) Misreading a question or answer.
Reason: This may occur because you overlooked a key word or phrase.
Suggestion: When choosing an answer, underline key words and phrases that stand out. Once you have underlined the
key words and phrases, check the details of the possible answers with the details you underlined, one by one. Always
keep in mind that you are looking for the best possible answer with the most accurate details.
3) Not knowing the meaning of key words.
Reason: This is a problem with vocabulary.
Suggestion: When you come to an unfamiliar word, reread the sentence to determine the general, overall meaning of
the sentence. This is known as “reading for meaning.” The meaning of the unfamiliar word should become clearer as you
begin to understand the phrases and ideas that surround it.
4) Answers that “look” good.
Reason: An incorrect answer may contain an exact phrase from the original question.
Suggestion: Beware of answers containing exact words or phrases from the question.
Reason: An incorrect answer may overstate information that was presented in the question. For example, if the question
says, “Some incidents…”, an incorrect answer may say, “All incidents…”
Suggestion: Stick strictly to the facts described in the exam question itself. Don’t be drawn to answers that stretch or
exaggerate these facts or rules. This is the time to watch out for words such as “only,” “never,” “always,” “whenever,”
“all,” etc.
General Test-Taking Tips
New York State Police Trooper Examination Study Guide 42
Copyright © 2022, IOS, Inc.
FINAL TIPS
Before the test
Prepare for the exam. You will increase your chances of obtaining your best score if you spend some time preparing
for the exam. This includes reading this study guide and finding out as much as possible about the exam ahead of
time, trying to avoid becoming overly anxious about the exam, and using good test-taking strategies.
On the Day of the Test
Allow yourself enough time for traveling to the test site and locating your test room.
Do NOT bring any device with a typewriter keyboard, such as a "Spell Checker," "Personal Digital Assistant (PDA),"
"Address Book," "Language Translator," "Dictionary," or other, similar device to the test site.
Do NOT bring cell phones, beepers, headphones, or any electronic or other communication devices to the test site.
The use of such devices at the test site in the test room, hallways, restrooms, building, grounds, or other areas could
result in your disqualification.
Do NOT bring books or reference materials to the test site.
Do NOT bring this guide or sample test materials to the test site.
During the Test
Read and follow all directions on your test administration materials, including candidate directions, test booklets, and
answer sheets.
Follow the test administrator’s instructions; raise your hand if you have questions or need help.
Keep track of the time and organize your work accordingly.
Do not look at the work of other people in the room or you may be disqualified.
After the Test
Continue to observe test security prohibitions by not removing any test materials from the test room, by not
reconstructing or reproducing test materials, and by not discussing the test.
Common Sense Suggestions
Get plenty of rest the night before.
Dress comfortably, in layers, so you can adapt to the temperature in the testing room.
Eat breakfast.
If you are not sure where the test site is, get directions to the site before the test date.
Leave yourself plenty of time to get to the test site just in case there is traffic, you have car trouble, transportation
problems, etc.
Your attitude and approach to the test will influence how well you perform. A positive attitude will help you do your
best.
Good luck on the examination and in your law enforcement career!
Practice Test
New York State Police Trooper Examination Study Guide 43
Copyright © 2022, IOS, Inc.
The NYSP Trooper Examination consists of 65 cognitive ability items and 85 items that measure personality attributes. On
the following pages, you will find a full 65-question practice test consisting entirely of cognitive ability questions.
Because you will have one hour and 40 minutes to complete the cognitive ability section of the actual exam, we
recommend that you spend no more than one hour and 40 minutes completing the practice test. This will ensure that you
can recreate the conditions of the real examination as accurately as possible. If possible, you should take this practice test
in a quiet room with few distractions. Additionally, you should complete the entire practice test before consulting the
answer key. In other words, the best way to prepare for the exam is to treat this practice test as if it were the real
examination.
AFTER TAKING THE PRACTICE TEST
Following the test will be an answer key so that you can check your answers. If you find that you have answered a question
incorrectly, go back to that question and try to figure out why you did not originally choose the correct answer. Make note
if you seem to be making the same types of mistakes repeatedly, then try to formulate a new strategy for answering those
questions. You may also wish to consult the tips listed in the study guide to improve your skills in each of the cognitive
ability areas tested to find a new question-answering strategy or to learn how to practice a certain skill.
ANSWER SHEET
You may mark your answers to the practice test questions on the following answer sheet. Upon completing the practice
test, compare your answers to the answer key on the last page.
1. A B C D
2. A B C D
3. A B C D
4. A B C D
5. A B C D
6. A B C D
7. A B C D
8. A B C D
9. A B C D
10. A B C D
11. A B C D
12. A B C D
13. A B C D
14. A B C D
15. A B C D
16. A B C D
17. A B C D
18. A B C D
19. A B C D
20. A B C D
21. A B C D
22. A B C D
23. A B C D
24. A B C D
25. A B C D
26. A B C D
27. A B C D
28. A B C D
29. A B C D
30. A B C D
31. A B C D
32. A B C D
33. A B C D
34. A B C D
35. A B C D
36. A B C D
37. A B C D
38. A B C D
39. A B C D
40. A B C D
41. A B C D
42. A B C D
43. A B C D
44. A B C D
45. A B C D
46. A B C D
47. A B C D
48. A B C D
49. A B C D
50. A B C D
51. A B C D
52. A B C D
53. A B C D
54. A B C D
55. A B C D
56. A B C D
57. A B C D
58. A B C D
59. A B C D
60. A B C D
61. A B C D
62. A B C D
63. A B C D
64. A B C D
65. A B C D
Practice Test
New York State Police Trooper Examination Study Guide 44
Copyright © 2022, IOS, Inc.
Answer Sheet
A
B
D
EXAMINATION INSTRUCTIONS
This examination consists of 65 multiple-choice questions. For each question, you must determine which one of the four
possible choices is the best answer. If you do not know the answer to a question, narrow your choices and select your
“best guess.” Your score on this examination will be the number of questions you answer correctly; no additional points
will be deducted for incorrect answers.
USING THE ANSWER SHEET
You have been provided with an answer sheet on which to record your response to each question. For each question,
various response options will be provided. Each question has only ONE correct answer. Please choose ONE response for
each question that best answers the question. Fill in your answer to each question by marking the appropriate “circle”
that corresponds to the question you are answering. Be sure that the question number you are answering matches the
question number on the answer sheet. You will be responsible for ensuring that your answers are marked in the
appropriate area.
If you wish to change an answer after marking the answer sheet, carefully erase the mark and blacken in the circle
corresponding to your final answer. Be careful not to blacken more than one answer for each question.
Following is an example of the correct method for answering exam questions:
1. The number of inches in one foot is __________.
a. 5 inches
b. 10 inches
c. 12 inches
d. 20 inches
There are 12 inches in one foot, so “c” is the correct answer to question one. You should indicate that “c” is the correct
answer by completely blackening the circle marked “c” on the answer sheet as displayed above.
Please answer all 65 questions. Your score on this examination will be the number of questions you answer correctly; no
additional points will be deducted for incorrect answers.
MEMORIZATION
You will begin with the memorization section. This section will involve two steps. Prior to beginning, you should set a timer
for six minutes. Once you turn the page, start the timer; you will be given six (6) minutes to study the three images
presented on the following three pages. You should memorize as many details as possible. After the six minutes expire,
you should turn the page. You will answer 10 questions about the memorization images (you should NOT flip back to look
at the images again after time has expired) and then continue with the rest of the questions in the practice test.
Start your six-minute timer and then turn the page.
Pr
ac
ti
ce
Te
st
Practice Test
New York State Police Trooper Examination Study Guide 45
Copyright © 2022, IOS, Inc.
Image 1: Crowd Control
Practice Test
New York State Police Trooper Examination Study Guide 46
Copyright © 2022, IOS, Inc.
Image 2: Boat
Practice Test
New York State Police Trooper Examination Study Guide 47
Copyright © 2022, IOS, Inc.
Image 3: Police Vehicles
Practice Test
New York State Police Trooper Examination Study Guide 48
Copyright © 2022, IOS, Inc.
This page intentionally left blank.
Practice Test
New York State Police Trooper Examination Study Guide 49
Copyright © 2022, IOS, Inc.
Use your memory of “Image 1: Crowd Control” to answer the following questions.
1.
What was the name of the street where the photo was taken?
a.
W 15th Street
b.
E 5th Avenue
c.
W 5th Street
d.
E 15th Street
2.
How many citizens in the photo were wearing a yellow shirt?
a.
One
b.
Two
c.
Three
d.
Four
3.
What was the name of the store in the background?
a.
AMERICAN EAGLE OUTFITTERS
b.
ABERCROMBIE & FITCH
c.
BROOKS BROTHERS
d.
AMERICAN WARDROBE
4.
What did the symbol on the street depict?
a.
A pedestrian
b.
A cyclist
c.
A one-way street
d.
A slow zone
Use your memory of “Image 2: Boat” to answer the following questions.
5.
How many people were visible on the boat?
a.
4
b.
5
c.
6
d.
7
6.
Which police department was inscribed on the side of the boat?
a.
New York
b.
Chicago
c.
San Francisco
d.
Salt Lake
Practice Test
New York State Police Trooper Examination Study Guide 50
Copyright © 2022, IOS, Inc.
7.
How many life preserver rings were visible in the photo?
a.
One
b.
Two
c.
Three
d.
Four
Use your memory of “Image 3: Police Vehicles” to answer the following questions.
8.
What was the license plate number of the closest vehicle in the photo?
a.
CC-112-IL
b.
CS-131-JL
c.
CR-121-IL
d.
CS-113-LJ
9.
How many police vehicles were visible in the photo?
a.
One
b.
Two
c.
Three
d.
Four
10.
How many fire trucks were visible in the photo?
a.
None
b.
One
c.
Two
d.
Three
Practice Test
New York State Police Trooper Examination Study Guide 51
Copyright © 2022, IOS, Inc.
Please use the passage to answer the questions that follow.
Last year, Brookcoast Police Department employees began noticing distrust among citizens in their community. Citizens
seemed fearful of officers and crime in the community was increasing. Despite the increased crime, citizens were not
reaching out to the police to share information or ask for help. Because of this, the department increased its focus on
community-oriented policing. Community-oriented policing relies on the community and police working together to
reduce crime. The Brookcoast Police Department implemented two new initiatives to help increase trust and create
partnerships with citizens in the community.
Department members first studied crime patterns in the community. They found that most crimes in Brookcoast last
year were committed by teenagers. To combat this, they designed a program for at-risk teens who have committed or
are likely to commit crimes. The program is six weeks long. It is hosted at the local community center. Officers volunteer
to serve as mentors to the teenagers and encourage them to get involved in community service activities. These
activities include serving food at the soup kitchen, walking dogs at the animal shelter, and cleaning up trash in the parks.
The aim of this program is to create trusting relationships with the community’s teenagers and to provide them with
productive ways to spend their free time.
While teens committed the majority of crimes in Brookcoast last year, statistics also showed that many citizens had
negative feelings towards the police. Because it is a high-crime area, many individuals have been arrested or know
others who have been arrested in the past. Therefore, the presence of an officer in the community was seen as negative.
To alleviate this negativity, the department started hosting a monthly “Coffee with a Cop” event. The event is hosted
in a local coffee shop. The department pays for food and drinks for citizens. This event allows citizens to talk to police
in a casual and comfortable environment. Its goal is to allow citizens to associate officers with positive experiences.
Brookcoast Police Department’s new programs have been very successful. Crime rates in the community have
decreased and communication between police and citizens is at an all-time high. Citizens no longer associate the police
with only negative experiences. Department members attribute the positive changes to the new partnerships with
community members that have been formed.
11.
Based on the passage, why did the department create the program for at-risk teens?
a.
To form positive relationships with them and provide them with things to do in their free time
besides committing crimes.
b.
To gather information from them about crimes they may have knowledge of and to make sure they
are not on the streets spreading fear.
c.
To punish them for committing the majority of crimes in the community and to give them better
things to do with their time.
d.
To help Brookcoast Police Department employees feel better about their relationships with
teenagers in the community.
12.
The main idea of the passage is _____.
a.
the Brookcoast Police Department struggled with fear and distrust among community members
until it began targeting at-risk teens
b.
community members will be more comfortable with police personnel if they host community
programs and provide them with free refreshments
c.
if a community is experiencing high levels of crime, it is important that community members
communicate any information they know to the police
d.
community-oriented policing initiatives can positively impact a community by decreasing crime and
improving relationships between citizens and the police
Practice Test
New York State Police Trooper Examination Study Guide 52
Copyright © 2022, IOS, Inc.
13.
Based on the passage, what effect did the monthly “Coffee with a Cop” event have on the community?
a.
Citizens began to associate the presence of police with positive experiences.
b.
The police became aware of the citizens’ negative feelings towards them.
c.
Citizens were able to talk to each other about their past experiences with the police.
d.
The police encouraged citizens to mentor at-risk teens in the community.
14.
The underlined word “alleviate” in the passage most closely means _____.
a.
to make worse
b.
to lessen
c.
to increase
d.
to relieve of duty
15.
The underlined word “committed” in the passage most closely means _____.
a.
pledged or bound
b.
carried out or perpetrated
c.
moved from one place to another
d.
feeling dedication and loyalty to a cause
Practice Test
New York State Police Trooper Examination Study Guide 53
Copyright © 2022, IOS, Inc.
Please use the passage to answer the questions that follow.
Drug usage and violence represent serious problems to both the community and law enforcement personnel.
Oftentimes, police departments must deal with sophisticated drug networks along with increasing gang violence that
accompanies the drug trade. To combat the problem, police departments perform many different activities. They
conduct drug educational programs in school, arrest drug users for illegal possession, and mount sting operations to
reduce open drug dealing, among other things. In addition, many law enforcement agencies also adopt alternative
strategies to curb the drug problem. Two such strategies are 1) “expressive law enforcement”, and 2) the “Mr. Big”
strategy.
Expressive law enforcement refers to simply increasing all current drug prevention activities by a factor of two or three.
For example, resources may be increased, special task forces created, or patrol units equipped and encouraged to make
more drug arrests. This strategy allows police departments to do what they do best: enforce the law. It also provides a
more targeted and direct attack on the problem that is easy to understand for both officers and community members.
However, a weakness of this strategy is that without an organized plan to give the community a role to play in the fight
on drugs, this strategy may actually deteriorate the self-defense capabilities of the community.
A second approach is the “Mr. Big” strategy. This strategy assumes it is more effective to target high-level drug
distributors rather than wasting resources on the arrest of low-level, easily replaced figures. Strengths of this strategy
are that it targets the criminal leaders whose greed, energy and intelligence sustain the drug trade. Also, the strategy
challenges police departments to improve their investigative and intelligence capabilities, leading to a more effective
police force. Weaknesses of this approach are that the resources used do not guarantee Mr. Big will be arrested. There
could also be many other potential Mr. Bigs ready to fill the role. Furthermore, the identity of Mr. Big may be unknown,
even to other dealers in the drug network.
While these strategies represent possible ways to combat drug issues in the community, perhaps the best approach is
one that is tailored to fit the specific needs of the community in question.
16.
Based on the passage, why might a Mr. Big strategy be effective in combatting drug problems?
a.
It targets those distributors who have the most control and influence over the drug distribution
network.
b.
The strategy forces a department’s narcotics and patrol divisions to work together more
collaboratively.
c.
Arresting high-level drug distributors creates fear among low-level figures and deters them from
selling drugs.
d.
The strategy sends a message to the community that the department is committed to solving the
root of the problem.
17.
The main idea of the passage is _____.
a.
despite their weaknesses, the two alternative strategies should be implemented in all police
departments
b.
the Mr. Big strategy could lead to a drain on department resources if police intelligence is not
credible
c.
due to the sophistication of criminal drug networks, police agencies are losing ground in the battle
against drugs
d.
several different strategies exist for combatting drug issues; agencies should adopt strategies to fit
their specific needs
Practice Test
New York State Police Trooper Examination Study Guide 54
Copyright © 2022, IOS, Inc.
18.
The underlined word “mount” in the passage most closely means _____.
a.
to arrange or assemble for display
b.
to seat or place oneself on
c.
to set or place at an elevation
d.
to organize and launch
For the following items, please choose the summary that describes the most important details in the scenario most
clearly and accurately.
19.
Police were dispatched to the scene of a traffic accident on the highway after a fuel truck flipped over and
burst into flames. A witness to the accident explained that he saw the truck swerve to avoid debris in the
road. He then witnessed the driver lose control of the truck while trying to correct the maneuver. The truck
flipped over onto its side, slid about 100 feet down the road, and then exploded. Debris was propelled into
the windshield of a sedan that was driving in the opposite direction. The truck driver was killed upon
impact and the driver of the sedan sustained minor injuries. The fire department was informed of the
accident and arrived shortly after the police.
a.
Two individuals were involved in an accident after a fuel truck swerved to avoid debris in the road
and exploded. The explosion caused damage to a sedan that was driving in the opposite direction.
b.
Police and firefighters responded to an accident on the highway. A fuel truck flipped over and
exploded after swerving to avoid debris on the road, killing its driver. Debris from the explosion hit
a sedan, resulting in minor injuries to its driver.
c.
An accident occurred while a fuel truck was driving on the highway. The truck flipped over and
exploded. Debris from the explosion hit the windshield of a sedan, injuring its driver. The fire
department arrived shortly after.
d.
Police were dispatched to the scene of an accident involving a fuel truck and a sedan. The driver of
the truck was killed upon impact and the driver of the sedan sustained minor injuries. The fire
department was dispatched to extinguish the fire.
Practice Test
New York State Police Trooper Examination Study Guide 55
Copyright © 2022, IOS, Inc.
20.
Shelley Holland calls 911 around 9:15 p.m. on Friday evening, and frantically tells the dispatcher that her
father, Brent Holland, is lying at the bottom of the stairs unresponsive. Dispatch sends troopers and medics
to the home. Upon arrival, the medics are able to revive Brent and transport him to the hospital. Trooper
Reeves interviews Shelley. She informs Trooper Reeves that upon entering her home, she noticed that the
drawers of several pieces of furniture were open; she then saw her father lying on the ground and called
911. Later, Trooper Reeves drives to the hospital to take a statement from Brent. Brent tells Trooper
Reeves that he was in the living room when he heard footsteps upstairs. He went to investigate and
encountered a strange man rifling through the dresser. The man saw Brent, pulled out a knife, and started
running towards him. Brent ran into the hallway and the man pushed him down the stairs. Brent says that
is the last thing he remembers before waking up in the hospital.
a.
Shelley Holland called 911 around 9:15 p.m. on Friday after arriving home, and realizing that her
home had been burglarized and her father was unconscious. Her father was taken to the hospital,
while Trooper Reeves interviewed her.
b.
Brent Holland was home alone when he heard footsteps upstairs. When he went to investigate, he
found a burglar rifling through drawers. The man attacked Brent, pushing him down the stairs. He
was knocked unconscious and later transported to the hospital.
c.
Shelley Holland called 911 on Friday after finding her father, Brent Holland, unresponsive. After
being revived by medics and transported to the hospital, Brent told Trooper Reeves he was pushed
down the stairs in his home after encountering a burglar upstairs.
d.
Brent Holland encountered a burglar in his home. The man chased Brent with a knife and then
pushed him down the stairs. His daughter called 911 after she arrived home and saw the drawers
of several pieces of furniture open.
21.
Troopers are dispatched to a senior living home after Ralph Williams calls 911 to complain that his
neighbors are being disruptive into the late hours of the night. Trooper Tucks and Trooper Kolb arrive at
Ralph’s apartment and Ralph opens the door, confused as to why the troopers are there. Trooper Kolb
explains that they are responding to his noise complaint. Ralph tells the troopers that he has not called
police recently, and proceeds to slam the door closed. Puzzled, Trooper Tucks knocks on the neighbor’s
door hoping to gain clarity regarding the situation. An elderly woman opens the door and explains that
Ralph has Alzheimer’s. When Trooper Kolb questions her about the noise complaint, the woman states
that she turns her TV volume up very loud because she has trouble hearing. She says she will make sure to
lower the TV volume at night. The troopers thank the woman for her cooperation and leave the building.
a.
Ralph’s neighbors were causing a disturbance, so he decided to call the police to complain. When
Trooper Tucks and Trooper Kolb arrived, they were puzzled when Ralph denied calling 911 and
slammed the door shut.
b.
Troopers responded to Ralph Williams’ noise complaint at a senior living facility and discovered that
he did not remember making the call. After speaking with his neighbor, they learned he has
Alzheimer’s. When questioned about the noise, the neighbor promised to lower her TV volume at
night.
c.
Trooper Kolb and Trooper Tucks were dispatched to a senior living facility to respond to a noise
disturbance call. The troopers spoke with the next-door neighbor, an elderly woman, who explained
the complainant has Alzheimer’s.
d.
An elderly man, Ralph Williams, called police to report a noise complaint; when troopers arrived,
Ralph denied calling 911 and slammed his door closed. The troopers spoke with the next-door
neighbor, who promised to turn down her TV at night to avoid another complaint.
Practice Test
New York State Police Trooper Examination Study Guide 56
Copyright © 2022, IOS, Inc.
22.
Trooper Ward responds to an emergency call at 8:15 a.m. to provide aid to Kevin Haden, who was jogging
along a state park trail when he was attacked by a dog. Kevin claims that he was about two hours into his
run when he saw the dog sprinting toward him. He noticed that the dog had a collar and dog tags, so he
assumed it would be friendly. However, as the dog approached him, it began barking, lunging, and biting
him. Kevin further states he is not sure how the attack unfolded, but the dog lunged at his legs several
times. Kevin tells Trooper Ward he climbed a nearby tree to escape from the dog, and it was at that time
he realized his legs were bleeding from several bites. The dog remained nearby for several minutes,
periodically barking, until a hiker chased the dog away with a large branch.
a.
Kevin Haden was jogging along a state park trail when a dog attacked. The dog displayed aggressive
behaviors. Kevin was only able to escape by climbing a tree. The dog remained in the area for several
minutes before being chased away.
b.
Kevin Haden was attacked by an aggressive dog on a state park trail. The dog had a collar and dog
tags. Kevin was able to escape by climbing a tree. The dog remained in the vicinity until it was chased
away by a hiker with a large branch.
c.
Kevin Haden was attacked by a dog while running on a state park trail. He was bitten several times
in the legs before he was able to escape by climbing a nearby tree. The dog was chased away by a
hiker with a large branch.
d.
While out for a 2-hour run along a state park trail, Kevin Haden was attacked by an aggressive dog.
Kevin stated that he had originally assumed the dog was friendly but ended up being bitten several
times in the legs.
23.
On Friday, June 13th, Trooper Pelham approaches a vehicle parked in the emergency lane on the interstate.
The sole occupant, Andrew Williams, claims that he was the victim of a hit-and-run. He explains that he
had an argument with his sister-in-law about an hour ago, which led to him leaving her house in his car to
“cool off.” As soon as he reached the interstate, he observed a white SUV rapidly approaching his vehicle.
The SUV got too close to the rear of Andrew’s vehicle, which resulted in a minor collision. The incident
ended when Andrew slowed his vehicle and the SUV subsequently sped away. Although Andrew was
unable to identify the driver of the SUV, he claims that his sister-in-law’s boyfriend drives a similar vehicle.
Additionally, Andrew believes his vehicle’s front-facing camera will have captured video footage of the
SUV as it sped away.
a.
After an argument with his sister-in-law, Andrew Williams was a victim of a hit-and-run when a
white SUV struck the back of his vehicle. Andrew was unable to identify the vehicle, but believes
that his vehicle may have footage of the SUV.
b.
Andrew Williams was the victim of a hit-and-run when another vehicle drove dangerously close to
him, causing a minor collision. Andrew stated that he had driven his car away from an argument
with his sister-in-law to “cool off.”
c.
Andrew Williams was involved in a minor collision after an argument with this sister-in-law. Andrew
stated a white SUV struck his vehicle from behind. Andrew also stated that his sister-in-law’s
boyfriend drives a similar vehicle.
d.
After an argument, Andrew Williams was involved in a minor car accident. Andrew stated that the
collision occurred shortly after reaching the interstate and that the other driver left the scene
shortly after.
Practice Test
New York State Police Trooper Examination Study Guide 57
Copyright © 2022, IOS, Inc.
For the following items, please choose the word or phrase that correctly completes the sentence.
24.
Police received complaints about a panhandler who was ______ pedestrians on the corner.
a.
harrasing
b.
harassing
c.
harrassing
d.
harasing
25.
Sergeant Brock checked with every trooper to ensure that everyone _______ understood the new policy.
a.
completely
b.
completly
c.
completaly
d.
compleatly
26.
Trooper Vaccaro examined the _______ wrist to see if a medic was needed.
a.
child
b.
child's
c.
childs'
d.
childs
27.
Sergeant Berry _____ that Trooper Spencer was going to be late for his shift because he informed him
ahead of time.
a.
new
b.
knowed
c.
knew
d.
knows
28.
Sergeant Brown will _____ to his shift thirty minutes early.
a.
arrive
b.
arrives
c.
arriving
d.
arrived
Practice Test
New York State Police Trooper Examination Study Guide 58
Copyright © 2022, IOS, Inc.
Identify the word that most closely matches the following definitions.
29.
Identify the word that has the most similar meaning to “retaliate.”
a.
Release
b.
React
c.
Provide
d.
Quarantine
30.
Identify the word that has the most similar meaning to “prohibit.”
a.
Endorse
b.
Discourage
c.
Permit
d.
Forbid
Practice Test
New York State Police Trooper Examination Study Guide 59
Copyright © 2022, IOS, Inc.
Please use the following information to answer the questions that follow.
The following tables summarize the license-suspension point system in the state of New York.
Maximum allowable points per calendar year (January 1 December 31):
Points
License Suspension Status
0 to 7 points
No suspension
8 to 12 points
Must complete Defensive Driving School*, or else
automatic 3 month license suspension
13 to 17 points
Automatic 3 month license suspension
18 to 23 points
Automatic 6 month license suspension
24+ points
Automatic 1 year license suspension
Point deductions by moving violation:
Category
Violation Type
Points
A
Failure to use turn signal
1
A
Careless driving
2
A
Improper lane switching
2
B
Failure to yield to pedestrians
2
B
Failure to yield to traffic signs or signals
3
C
Failure to yield to emergency or public vehicles
3
B
Excessive speed: 1-14 MPH over posted speed limit
3
B
Excessive speed: 15-20 MPH over posted speed limit
4
C
Excessive speed: 21+ MPH over posted speed limit
5
C
Reckless driving
6
C
Causing accident with injuries
8
B
Failure to use a seat belt
8
C
Leaving the scene of accident; hit-and-run
9
C
Driving under the influence
10
*The following rules apply to motorists completing Defensive Driving School:
Defensive Driving School is only applicable to Category A and Category B violations.
Defensive Driving School does not remove points from a motorist’s record.
In order to be eligible, motorist must not have completed Defensive Driving School in the 12 months prior to
the current violation date.
Motorists are only eligible to take Defensive Driving School for one moving violation at a time.
If the motorist possesses a commercial driver’s license (CDL), he/she is not eligible for Defensive Driving
School, regardless of the category of moving violation committed.
Practice Test
New York State Police Trooper Examination Study Guide 60
Copyright © 2022, IOS, Inc.
31.
Molly Hanson last completed a Defensive Driving School course for a violation she received on April 1,
2019. When is Molly next eligible to complete Defensive Driving School?
a.
Any violation occurring on or after April 1, 2020.
b.
Any violation occurring on or after October 1, 2019.
c.
Any violation occurring on or after June 1, 2019.
d.
Any violation occurring on or after October 1, 2020.
32.
Antonio De Julio was first cited for speeding on March 10, going 16 MPH above the posted speed limit. On
May 29, Antonio is cited again for leaving the scene after getting into a car accident. Given this information,
what penalty will Antonio face?
a.
Antonio must complete Defensive Driving School, or else his license will be suspended for three
months.
b.
Antonio’s license will be suspended for six months.
c.
Antonio will not receive any license suspension.
d.
Antonio’s license will be suspended for three months.
33.
Tomasz Wardzala is a truck driver with a commercial driver’s license. On September 9, Tomasz was
returning from a friend’s home when his vehicle was randomly selected to participate in a police sobriety
checkpoint. Officer Mason administered the road-side sobriety check and found Mr. Wardzala to be within
legal limits. Officer Mason did however notice that Mr. Wardzala was not wearing his seatbelt and
therefore cited him for the seatbelt violation. While writing the citation, Officer Mason noted this was his
first moving violation in eight years. Based on this information, which of the following statements is
correct?
a.
Since this is Mr. Wardzala’s first moving violation in eight years, he is eligible for Defensive Driving
School. He will avoid having his license suspended for a period of three months.
b.
Since this moving violation is a Category B offense, Mr. Wardzala is not eligible for Defensive Driving
School. His license will be suspended for a period of three months.
c.
Since Mr. Wardzala has a commercial driver’s license, he is not eligible for Defensive Driving School.
His license will be suspended for a period of six months.
d.
Since Mr. Wardzala has a commercial driver’s license, he is not eligible for Defensive Driving School.
His license will be suspended for a period of three months.
Practice Test
New York State Police Trooper Examination Study Guide 61
Copyright © 2022, IOS, Inc.
Please use the following information to answer the questions that follow.
The following is an excerpt from department’s time off policy:
Paid Time Off:
A. Paid time off will include the following:
Vacation, personal, floating holiday, compensatory, and extended leave
To the best of the employee’s ability, all paid time off shall be scheduled in advance.
B. Unscheduled paid time off will include the following:
Shift employees: paid time off not scheduled at least 24 hours in advance
Staff employees: paid time off not scheduled at least 10 hours in advance
C. Leave Abuse Occurrences
The recurrent use of unscheduled leave
Using both unscheduled leave with scheduled leave simultaneously
Use of unscheduled or sick leave for other employment
D. Disciplinary action will be taken for the accumulation of leave abuse occurrences in a 1-year period:
3
rd
occurrence: documented consultation and review of attendance policy
4
th
occurrence: written reprimand
5
th
occurrence: 24-hour to 36-hour suspension
6
th
occurrence: termination of employment
Sick Leave:
A. Sick leave will include the following:
When an employee has an illness or disability that would prevent them from performing the duties of the
police department as assigned
The employee shall notify the Shift Commander of absence due to illness as soon as possible, but no later
than one-half (1/2) hour before the start of the employee’s work shift.
Employees are allocated seven sick days per year.
Failure to properly report an illness may be considered as unscheduled time off.
B. Medical Examination
The Shift Commander may require an employee to submit a physician’s verification of illness.
If the employee will be absent for three consecutive shifts, the Shift Commander may also require a
physician’s verification that the employee is well enough to return to work.
C. Medical Verifications must contain the following information:
The name of the physician and diagnosis
The date of the examination and the date the employee is allowed to return to duty
Must state if there are any work restrictions
Practice Test
New York State Police Trooper Examination Study Guide 62
Copyright © 2022, IOS, Inc.
34.
During the summer, Trooper Benson picks up shifts as a lifeguard on his days off. Recently there has been
a shortage of lifeguards. Trooper Benson decides to call in sick to his supervisor two hours before the start
of his shift so that he can work more lifeguarding shifts. In total, he has taken five sick days this year. Is
Trooper Benson using sick time appropriately?
a.
Yes, Trooper Benson can use his sick time as he sees appropriate.
b.
No, Trooper Benson did not notify the department in a timely manner.
c.
No, Trooper Benson is only permitted to use sick leave when he has an illness.
d.
No, Trooper Benson has exceeded the number of allotted sick days.
35.
Trooper Kessler goes to his physician for symptoms of bronchitis. Which of the following statements is
TRUE regarding how Trooper Kessler should document his illness?
a.
Trooper Kessler will need to notify the police department of his illness at least 24 hours before his
next shift.
b.
Trooper Kessler’s medical verification must report the date he first started showing symptoms.
c.
Trooper Kessler will need verification that he is well enough to work if he misses more than one
scheduled shift.
d.
Trooper Kessler’s medical verification must report if he will have work restrictions from his illness.
Please use the passage below to answer the question that follows.
Investigator Jensen interviewed an eyewitness to a robbery. The following statements appear in his report about the
interview. The statements are not in order.
1. Approximately ten minutes later, at about 12:30, the two people ran out of the bank carrying duffel bags and
got back into the van.
2. His friend had just arrived around 12:15 when a black van drove up to the bank beside the restaurant.
3. Shortly afterwards, at about 12:40, three police cruisers arrived at the bank with flashing lights and sirens.
4. Two people wearing masks and black clothing exited the van and entered the bank.
5. The black van quickly drove away from the bank.
6. The eyewitness was eating lunch at an Italian restaurant around 12:00.
36.
Which of the following is the correct order of the above statements?
a.
6, 2, 4, 1, 5, 3
b.
6, 4, 2, 1, 5, 3
c.
2, 6, 4, 1, 5, 3
d.
6, 2, 4, 1, 3, 5
Practice Test
New York State Police Trooper Examination Study Guide 63
Copyright © 2022, IOS, Inc.
Please use the passage below to answer the question that follows.
Iris Goodman reported to police that someone had been using her debit card fraudulently. Her account of the incident
consists of the following six statements. The statements are not in order.
1. I called my husband to ask if he had made the purchases, and he said he was nowhere near Oakville yesterday.
I wasn’t either.
2. When I got home, I logged in to my bank account to determine why my card was declined.
3. I called my bank and they deactivated my debit card. They told me I should receive a new one in the mail
tomorrow.
4. I was so confused. I saw numerous transactions that had occurred in Oakville yesterday, and my account
balance was negative!
5. They also told me that I should file a police report regarding the fraud. I hope you can find who did this!
6. This morning, I tried to use my debit card at the grocery store and the purchase was declined. I had to use a
different card because the transaction wouldn’t go through.
37.
Which of the following is the correct order of the above statements?
a.
6, 2, 4, 1, 3, 5
b.
6, 2, 1, 4, 3, 5
c.
6, 3, 2, 1, 5, 4
d.
6, 1, 2, 4, 5, 3
Please use the passage below to answer the question that follows.
Below is a sample of a report given to police by a witness. The statements are not in order.
1. As I walked away from my car, I noticed one other car in the parking lot. It was dark, so I couldn’t tell what
kind of vehicle it was.
2. While I waited for the police to arrive, I wandered around the parking lot. I found my purse discarded in a
nearby recycling can.
3. When I looked inside, I found that my purse and sunglasses were missing. All the compartments in my vehicle
were opened; luckily, there was nothing else of value in the car.
4. About an hour later, I returned to my car and immediately noticed that the driver’s side window had been
smashed.
5. The whole event shook me up quite a bit, so I had to sit down and take a few deep breaths before I called the
police.
6. I left my apartment around 6:45 a.m., drove to the park, and parked my car in the west parking lot.
38.
Which of the following is the correct order of the above statements?
a.
6, 1, 4, 3, 5, 2
b.
6, 4, 3, 2, 1, 5
c.
6, 3, 2, 1, 4, 5
d.
6, 1, 4, 5, 2, 3
Practice Test
New York State Police Trooper Examination Study Guide 64
Copyright © 2022, IOS, Inc.
Please use the passage below to answer the question that follows.
Below is a sample of a report given to police by a witness. The statements are not in order.
1. I asked him what he was looking at. He then approached me and punched me in the nose before running away
across the parking lot.
2. The man said he saw me chasing after him with blood all over my shirt and wanted to help. He dialed 911
from the car and reported our location.
3. I chased him across the parking lot and down Main Street for about a block. He then ran into this neighborhood
but I didn’t see where he went. He might have jumped a fence.
4. I was walking into the Deluxe Market to pick up groceries when I saw this man out of the corner of my eye.
5. A few minutes later, we saw a squad car arrive, and I got out of the truck to wave the officers down.
6. He was standing next to one of the cart return corrals while staring at me in a very strange way.
7. I was jogging down the street looking for where he might have gone, when a man in a pickup truck yelled to
me to get in so we could look for him.
8. I stumbled a bit and clutched my nose, feeling the blood pouring out. Then he started running away, and I
started chasing after him.
39.
Which of the following is the correct order of the above statements?
a.
4, 6, 1, 8, 3, 2, 7, 5
b.
4, 6, 1, 8, 3, 7, 2, 5
c.
4, 6, 1, 3, 8, 7, 2, 5
d.
4, 6, 1, 8, 3, 7, 5, 2
Please use the passage below to answer the question that follows.
Allison Burns was in a traffic accident involving three vehicles. Her description of the event includes the following six
statements. The statements are not in order.
1. She ignored me and continued texting. As she was looking down at her phone, the car in front of us slammed
on its brakes.
2. Sarah and I got out of the car to make sure everyone was okay. The other two drivers were angry, but no one
was injured.
3. We were almost to the restaurant when Cindy texted Sarah and me saying that she would be late.
4. Sarah picked me up from my house at 11:30 because we were going to meet Cindy at a restaurant for lunch.
5. Sarah did not notice the stopped car and she rear-ended it. The car behind us did not stop in time, and it hit
Sarah’s car.
6. Sarah pulled her cell phone out of her pocket and started to text Cindy back. I told her to put her phone down
because she was driving.
40.
Which of the following is the correct order of the above statements?
a.
3, 4, 5, 6, 1, 2
b.
4, 3, 6, 1, 5, 2
c.
4, 3, 5, 6, 1, 2
d.
3, 5, 4, 6, 1, 2
Practice Test
New York State Police Trooper Examination Study Guide 65
Copyright © 2022, IOS, Inc.
Please use the information presented to answer the following questions.
Police respond to a mugging in the 5400 block of West Parker Avenue. Four individuals provide information to the police
once they arrive on scene at approximately 8:55 a.m.
Chuck Yeager: “It happened pretty early in the morning, around 8:15 a.m. I can’t believe somebody had the audacity
to mug Emily while she was clearing snow off her Buick Regal. She has lived across the street from me for at least twenty
years. Even though it happened right across the street from where I live, I didn’t see it happen, but I heard her yelling
outside which prompted me to come downstairs. She mentioned that a young kid, high school-aged, just snatched her
purse and sped off toward the nearest busy street, which is Diversey Avenue, just north of our block. I tried to track the
kid down but I couldn’t find anyone when I walked down the street.”
Clint McThomas: “The snow has been so intense over the last twelve hours! I found my car in the morning under a
mountain of snow at the end of the block. I was shoveling my Ford Bronco out around 8:00 a.m. when I saw someone
run up to Emily and then quickly run away. It was hard to make out the details from half a block away, but it appeared
to be a young person wearing a black hooded sweatshirt with a ski mask and some black pants. He was running in the
opposite direction of where my car was, and it all happened so fast.”
Tina Rogers: “I awoke around 8:00 a.m. to yelling outside my apartment complex. When I looked outside my bedroom
window I saw an individual holding a black purse in his right hand and running toward Diversey Avenue. He ran almost
an entire block from the mugging location to where I live before I caught a glimpse of his face as he turned the corner,
where a silver Chevrolet Malibu with tinted windows was waiting. The suspect, wearing gray pants, jumped into the
front passenger seat and the car immediately drove off. The whole thing must have been planned, as the timing was
just too perfect. A couple of minutes later I saw my neighbor, Chuck, walking down the exact same route that the person
first ran down. By that time however, the car was long gone. When I came downstairs and talked with Emily, she was
so scared; the suspect must have been armed!”
Samantha Brule: “I work evenings as a nurse at a hospital a few blocks away. Even though it’s so close, I always drive
because the neighborhood can get very dangerous during the night. My husband, who was driving to work, called me
around 9:00 a.m. to tell me something had happened about two blocks from where we live and some police were on
scene. I quickly threw on some clothes and went to go investigate for myself, but had no details to give to police.”
41.
Based on the witness statements, which of the following conclusions is true?
a.
Emily was mugged at gunpoint.
b.
The suspect had an accomplice.
c.
The suspect was wearing black jeans.
d.
Emily was mugged around 7:00 AM.
42.
Rank order the proximity from closest to furthest of each witness from Emily at the time of the mugging.
a.
Chuck, Clint, Tina, Samantha
b.
Chuck, Tina, Clint, Samantha
c.
Clint, Chuck, Tina, Samantha
d.
Tina, Chuck, Clint, Samantha
43.
Which witness would be best able to identify the suspect in a lineup?
a.
Clint McThomas
b.
Chuck Yeager
c.
Samantha Brule
d.
Tina Rogers
Practice Test
New York State Police Trooper Examination Study Guide 66
Copyright © 2022, IOS, Inc.
Please use the information presented to answer the following questions.
Troopers have been dispatched to a high school to investigate the discovery of marijuana in one of the school’s male
restrooms. The school’s administration has seen a surge of students using the drug. Troopers interview four witnesses
concerning the drug. Their accounts are below.
Witness 1: “I have been a technology teacher here for ten years. My classroom is located right next to restrooms on
the west end of the school. One student, Harry Baldwin, passed my classroom five times this morning. I’m not sure if
he was going to the men’s bathroom, but I’m not sure where else he would have been going because my classroom is
close to the end of the hallway, near the rear parking lot. There are doors leading to the parking lot, but students are
not permitted to use those doors. He definitely looked suspicious.”
Witness 2: “I work for a maintenance company that fixes kitchen equipment. I had just finished fixing an oven in the
cafeteria at 8:20 a.m., which is on the east end of the school, when two teenagers walked past me, smelling weird and
acting suspicious. When they saw me, they got really nervous. One of the students was wearing a school athletic shirt
that said “Jones” on the back. I’m assuming that was his last name. I heard him say, ‘See you later, Tommy’ to the other
student. I didn’t see any staff members because there are no classrooms on this end of the school, so I didn’t tell
anybody. I guess I could have checked if the PE teacher was in the gym since it is about 50 feet west of the cafeteria.
Witness 3: “I’m the track coach for the junior varsity team. Our team has practice every Monday at 8:45 a.m. On this
particular day, one of my athletes, Brady Jones, arrived five minutes late and appeared ‘off’ during practice. I called him
over and asked if everything was alright. He had bloodshot eyes and smelled weird but insisted he just felt sick. I asked
if he wanted to go see the nurse, but he appeared to become nervous and instead asked to sit out the rest of practice.
I thought it was odd but didn’t think much else of it.
Witness 4: “I am a wood shop teacher here. I have heard rumors that a student is selling marijuana. There is only one
set of bathrooms in the hallway where my classroom is located; they are at the end of the hallway near the technology
classroom. The only student I saw this morning was my teaching assistant, Harry. Around 8:10 a.m. I had him unload a
truck with wood shop supplies. It took him about an hour and a half to get everything unloaded. Without a teacher’s
permission, students are not allowed to exit out the doors, but he was helping me.”
44.
Based on the witness statements, order the school locations from closest to the restrooms to furthest from
the restrooms.
a.
Technology classroom, gym, cafeteria, wood shop
b.
Wood shop, gym, technology classroom, cafeteria
c.
Cafeteria, gym, wood shop, technology classroom
d.
Technology classroom, wood shop, gym, cafeteria
45.
Which of the following statements is TRUE?
a.
The track coach and the wood shop teacher both interacted with Harry.
b.
Harry is selling marijuana to students.
c.
Harry is not on the track team.
d.
The maintenance man saw Harry with Brady.
Practice Test
New York State Police Trooper Examination Study Guide 67
Copyright © 2022, IOS, Inc.
For the following items, please select the answer that completes the pattern.
46.
a.
b.
c.
d.
47.
a.
b.
c.
d.
Practice Test
New York State Police Trooper Examination Study Guide 68
Copyright © 2022, IOS, Inc.
48.
a.
b.
c.
d.
49.
a.
b.
c.
d.
Practice Test
New York State Police Trooper Examination Study Guide 69
Copyright © 2022, IOS, Inc.
50.
a.
b.
c.
d.
For the following questions, images of faces are presented. You will be asked to select an identical face from among
a series of four faces to which various manipulations have been made (e.g., hair styles, image angles, facial
expressions, etc.).
51.
Which of the images below is the same woman shown here, but from a side
profile?
a.
b.
c.
d.
Practice Test
New York State Police Trooper Examination Study Guide 70
Copyright © 2022, IOS, Inc.
52.
Which of the images below is the same woman shown here, but from a side
profile?
a.
b.
c.
d.
53.
Which of the images below is the same woman shown here, but with a
happy facial expression?
a.
b.
c.
d.
Practice Test
New York State Police Trooper Examination Study Guide 71
Copyright © 2022, IOS, Inc.
54.
This man has shaved his head. Which of the images below is the same man, but
with no hair?
a.
b.
c.
d.
55.
This woman has changed her hair style. Which of the images below is the same
woman, but with a different hair style?
a.
b.
c.
d.
Practice Test
New York State Police Trooper Examination Study Guide 72
Copyright © 2022, IOS, Inc.
This page intentionally left blank.
Practice Test
New York State Police Trooper Examination Study Guide 73
Copyright © 2022, IOS, Inc.
Please use the map below to answer the following questions. Assume that all streets are two-way streets.
56.
You are at the intersection of Cedar Street and North Forest Avenue. You drive the following route: north
on North Forest Avenue, west on Alder Street, north on North Ella Avenue, east on Hickory street, and
north on North Forest Avenue, and stop at the first intersection. Which of the following intersections is
closest to your new position?
a.
Chestnut Street and North Ruth Avenue
b.
Larch Street and North Forest Avenue
c.
North Ruth Avenue and Spruce Street
d.
North Boyer Avenue and Larch Street
N
Practice Test
New York State Police Trooper Examination Study Guide 74
Copyright © 2022, IOS, Inc.
57.
You are at the intersection of North Lincoln Street and Poplar Street. You drive the following route: east
on Poplar Street, north on North Monroe Avenue, southeast on Main Street, north on Washington Avenue
and west on Hickory Street, and stop at the first intersection. Which of the following intersections is closest
to your new position?
a.
Spruce Street and North Jefferson Avenue
b.
North Ella Avenue and Larch Street
c.
N Division Avenue and Walnut Street
d.
Fir Street and North Jefferson Avenue
58.
You are at the intersection of North Florence Avenue and Larch Street. You drive the following route: south
on North Florence Avenue, west on Alder Street, north on N Division Avenue, and east on Walnut Street,
and stop at the first intersection. Which of the following intersections is closest to your new position?
a.
Chestnut Street and North Jefferson Avenue
b.
Washington Avenue and Hickory Street
c.
Spruce Street and N Division Avenue
d.
North Ella Avenue and Chestnut Street
Practice Test
New York State Police Trooper Examination Study Guide 75
Copyright © 2022, IOS, Inc.
Please use the map below to answer the following questions. Assume that all streets are two-way streets.
N
Practice Test
New York State Police Trooper Examination Study Guide 76
Copyright © 2022, IOS, Inc.
59.
You are at the intersection of West Sunset Street and South Campbell Avenue. You drive the following
route: north on South Campbell Avenue, east on West Portland Street, south on South Dollison Avenue,
west on East University Street, and stop at the first intersection. Which of the following intersections is
closest to your new position?
a.
East Cherokee Street and South Maryland Avenue
b.
East Cherokee Street and South Roanoke Avenue
c.
East Kingsbury Street and South Kings Avenue
d.
East Seminole Street and South Holland Avenue
60.
You are at the intersection of West Tracy Street and South Campbell Avenue. You drive the following route:
south on South Campbell Avenue, east on West Sunset Street, north on South Jefferson Avenue, east on
East Crestview Street, and stop at the second intersection. Which of the following intersections is closest
to your new position?
a.
East Seminole Street and South Maryland Avenue
b.
East Downing Street and South Virginia Avenue
c.
East Wayland Street and South Campbell Avenue
d.
West Silsby Street and South Campbell Avenue
Practice Test
New York State Police Trooper Examination Study Guide 77
Copyright © 2022, IOS, Inc.
Please use the image below to answer the following questions.
61.
Which of the following license plate numbers appears in the above image?
a.
JST 8206
b.
JPS 8882
c.
BAV 3305
d.
BCR 1521
62.
In the image above, which license plate contains the plate number “BEA 6222”?
a.
b.
c.
d.
63.
Which of the following license plate numbers appears in the above image?
a.
DBG 8448
b.
BAS 8532
c.
DGB 8484
d.
JOR 8700
Practice Test
New York State Police Trooper Examination Study Guide 78
Copyright © 2022, IOS, Inc.
This page intentionally left blank.
Practice Test
New York State Police Trooper Examination Study Guide 79
Copyright © 2022, IOS, Inc.
Please use the image below to answer the following questions.
64.
How many times does the following face appear?
a.
One time
b.
Two times
c.
Three times
d.
Four times
Practice Test
New York State Police Trooper Examination Study Guide 80
Copyright © 2022, IOS, Inc.
65.
How many times does the following face appear?
a.
One time
b.
Two times
c.
Three times
d.
Four times
Practice Test
New York State Police Trooper Examination Study Guide 81
Copyright © 2022, IOS, Inc.
ANSWER KEY
Review your answers to the practice questions and use the table below to indicate if your answer was correct or incorrect
by making a check or X mark in the proper column. When you are finished, move on to the “Evaluating Your Performance”
section on the following pages for guidance in calculating and interpreting your scores.
#
Cognitive Ability
Answer
Correct/
Incorrect
1
Memorization
D
2
Memorization
B
3
Memorization
A
4
Memorization
B
5
Memorization
B
6
Memorization
B
7
Memorization
B
8
Memorization
D
9
Memorization
C
10
Memorization
B
11
Reading
A
12
Reading
D
13
Reading
A
14
Vocabulary
B
15
Vocabulary
B
16
Reading
A
17
Reading
D
18
Vocabulary
D
19
Writing
B
20
Writing
C
21
Writing
B
22
Writing
C
23
Writing
A
24
Grammar
B
25
Grammar
A
26
Grammar
B
27
Grammar
C
28
Grammar
A
29
Vocabulary
B
30
Vocabulary
D
31
Deductive Reasoning
A
32
Deductive Reasoning
D
33
Deductive Reasoning
D
#
Cognitive Ability
Answer
Correct/
Incorrect
34
Deductive Reasoning
C
35
Deductive Reasoning
D
36
Ordering of Information
A
37
Ordering of Information
A
38
Ordering of Information
A
39
Ordering of Information
B
40
Ordering of Information
B
41
Inductive Reasoning
B
42
Inductive Reasoning
A
43
Inductive Reasoning
D
44
Inductive Reasoning
D
45
Inductive Reasoning
C
46
Pattern Recognition
C
47
Pattern Recognition
B
48
Pattern Recognition
B
49
Pattern Recognition
C
50
Pattern Recognition
B
51
Visualization
A
52
Visualization
C
53
Visualization
D
54
Visualization
B
55
Visualization
D
56
Spatial Orientation
C
57
Spatial Orientation
A
58
Spatial Orientation
A
59
Spatial Orientation
C
60
Spatial Orientation
B
61
Flexibility of Closure
D
62
Flexibility of Closure
B
63
Flexibility of Closure
C
64
Flexibility of Closure
D
65
Flexibility of Closure
C
Practice Test
New York State Police Trooper Examination Study Guide 82
Copyright © 2022, IOS, Inc.
EVALUATING YOUR PERFORMANCE
The following tables will assist you in calculating percentage scores for the 12 cognitive ability sections as well as for your
overall score. First, add up the total number of items you answered correctly for each section. Then, add up the number
of items you answered correctly on the test as a whole. Write these numbers in the corresponding spaces in Table 1 below.
Table 1: Section and Overall Scores
Items
Section
Number Correct
Percentage Score
1-10
Memorization
____ / 10
11-13, 16-17
Reading
____ / 5
14-15, 18,
29-30
Vocabulary
____ / 5
19-23
Writing
____ / 5
24-28
Grammar
____ / 5
31-35
Deductive Reasoning
____ / 5
36-40
Ordering of Information
____ / 5
41-45
Inductive Reasoning
____ / 5
46-50
Pattern Recognition
____ / 5
51-55
Visualization
____ / 5
56-60
Spatial Orientation
____ / 5
61-65
Flexibility of Closure
____ / 5
1-65
Overall Score
____ / 65
Next, use Table 2 below to obtain your percentage score for the memorization section. Write this percentage score in the
appropriate space in Table 1.
Table 2: Memorization Section Percentage Score Calculation
# Correct
Memorization Section
Percentage Score
10 correct
100%
9 correct
90%
8 correct
80%
7 correct
70%
6 correct
60%
5 correct
50%
4 correct
40%
3 correct
30%
2 correct
20%
1 correct
10%
Now, use Table 3 to obtain your percentage score for each of the other sections (all besides Memorization). Write these
percentage scores in the appropriate spaces in Table 1.
Practice Test
New York State Police Trooper Examination Study Guide 83
Copyright © 2022, IOS, Inc.
Table 3: Other Section Percentage Score Calculations
# Correct
Section Percentage Score
5 correct
100%
4 correct
80%
3 correct
60%
2 correct
40%
1 correct
20%
If you achieved a score of 70% to 100% on a section, your ability in this area is high. It would still be beneficial for you to
spend additional time practicing in areas where your score was below 100%.
If your score was 60% or lower in any section, this represents a deficiency that should be addressed with continued
study and practice. An ability level at or below the 60% level will likely result in poor performance on the examination.
Finally, you can use Table 4 below to obtain your overall percentage score. Write this percentage score in the
appropriate space in Table 1.
Table 4: Overall Percentage Score Calculation
# Correct
Overall
Percentage
Score
65
100%
64
98%
63
97%
62
95%
61
94%
60
92%
59
91%
58
89%
57
88%
56
86%
55
85%
54
83%
53
82%
52
80%
51
78%
50
77%
49
75%
48
74%
47
72%
46
71%
45
69%
44
68%
# Correct
Overall
Percentage
Score
43
66%
42
65%
41
63%
40
62%
39
60%
38
58%
37
57%
36
55%
35
54%
34
52%
33
51%
32
49%
31
48%
30
46%
29
45%
28
43%
27
42%
26
40%
25
38%
24
37%
23
35%
22
34%
# Correct
Overall
Percentage
Score
21
32%
20
31%
19
29%
18
28%
17
26%
16
25%
15
23%
14
22%
13
20%
12
18%
11
17%
10
15%
9
14%
8
12%
7
11%
6
9%
5
8%
4
6%
3
5%
2
3%
1
2%
After you determine your overall percentage score, refer to the Score Interpretation Guide on the following page to gain
further insight into your score.
Practice Test
New York State Police Trooper Examination Study Guide 84
Copyright © 2022, IOS, Inc.
Table 5: Overall Score Interpretation Guide
Your Score
Category
100%- 80%
Strong: Your strong practice test score indicates that you are likely to perform well on the
cognitive section of the examination. Stay on track and continue studying and practicing.
Review the test-taking tips in the study guide to ensure that you are at your best when you
take the actual examination. Focus on any “weaker” dimensions to enhance your score.
79%- 61%
Approaching Readiness: Your score indicates that you are close to being on track to perform
well on the examination, but you need to continue to strengthen your skills. To increase your
readiness, review Table 1 and determine any sections in which you performed poorly. Review
these sections in both the study guide and the practice test.
60%- 0%
Needs Improvement: Your score indicates that you require more practice. Review Table 1 and
determine which sections you performed poorly in. Continue to review the study guide and
the practice test to improve your proficiency in each area.
If you missed questions in any cognitive ability area, you should continue practicing that ability area. The cognitive section
of the test is your best opportunity to improve your test score every question you get correct could substantially improve
your ranking.
Industrial/Organizational Solutions, Inc.
Public Safety Selection Experts
Copyright © 2020. All rights reserved.
Industrial/Organizational Solutions, Inc.